ChaseDream

标题: 【每日逻辑练习第二季】【1-19】 [打印本页]

作者: balapupu    时间: 2011-12-7 21:28
标题: 【每日逻辑练习第二季】【1-19】
今天记得发作业了~~~嘿嘿~~~今天把daisy&bat整理的bible note 看了~~真的是很好的材料哈~~大家木有时间看那个厚的可以看这个~~不知不觉我们已经做完了旧prep的第一部分啦~~各位继续加油哈~~
【精练】


6. Unlike newspapers in the old days, today’s
newspapers and televised news programs are full of
stories about murders and assaults in our city. One
can only conclude from this change that violent
crime is now out of control, and, to be safe from
personal attack, one should not leave one’s home
except for absolute necessities.
Which one of the following, if true, would cast the
most serious doubt on the conclusion?
(A) Newspapers and televised news programs have
more comprehensive coverage of violent
crime than newspapers did in the old days.
(B) National data show that violent crime is out of
control everywhere, not just in the author’s
city.
(C) Police records show that people experience
more violent crimes in their own
neighborhoods than they do outside their
neighborhoods.
(D) Murder comprised a larger proportion of
violent crimes in the old days than it does
today.
(E) News magazines play a more important role
today in informing the public about crime
than they did in the old days.


【逻辑链】
1.    (24021-!-item-!-188;#058&000691)


On Pacific islands, a newly arrived gecko species, the house gecko, is displacing the previously established mourning gecko in urban areas, but populations of the two species are more stable in rural areas far from human settlement.  The house gecko does not attack the mourning gecko, but in areas where insects congregate it prevents the mourning gecko from feeding on them.


Which of the following contributes most to an explanation of the difference between gecko populations in urban and rural areas?


(A) In urban areas, geckos are valued because they eat so many insects.
(B) Geckos defend territories against other members of the same species.
(C) House geckos that arrive on islands are carried there in boats and planes.
(D) In places where there are lighted buildings, insects tend to gather around the light.
(E) Mourning geckos are all females and reproduce asexually, but house geckos reproduce sexually.


2.    (24069-!-item-!-188;#058&000703)


Downtown Villieu was once a flourishing business district, but most Villieu-area businesses are now located only in the suburbs.  The office buildings downtown lack the modern amenities most business operators demand today.  To lure more businesses downtown, Villieu officials plan to have several modern office buildings constructed and to offer reduced local tax rates to any business that leases space in a new downtown building.


Which of the following, if true, most threatens the plan's likelihood of success?


(A) Most of the businesses currently located in downtown Villieu have long-term leases on the space they occupy there.
(B) The existing office buildings in downtown Villieu have, on average, a much higher vacancy rate than do office buildings in Villieu's suburbs.
(C) The local tax rates in Villieu's suburbs are significantly lower than downtown Villieu's proposed rate for businesses that lease space in the new office buildings.
(D) Most of the businesses that currently lease office space in downtown Villieu also lease office space in Villieu's suburbs.
(E) With the new office buildings, downtown Villieu would have a greater amount of modern office space than any other downtown business district in the region.


3.    (24117-!-item-!-188;#058&000708)


Northern Air has dozens of flights daily into and out of Belleville Airport, which is highly congested.  Northern Air depends for its success on economy and quick turnaround and consequently is planning to replace its large planes with Skybuses, whose novel aerodynamic design is extremely fuel efficient.  The Skybus' fuel efficiency results in both lower fuel costs and reduced time spent refueling.


Which of the following, if true, could present the most serious disadvantage for Northern Air in replacing their large planes with Skybuses?


(A) The Skybus would enable Northern Air to schedule direct flights to destinations that currently require stops for refueling.
(B) Aviation fuel is projected to decline in price over the next several years.
(C) The fuel efficiency of the Skybus would enable Northern Air to eliminate refueling at some of its destinations, but several mechanics would lose their jobs.
(D) None of Northern Air's competitors that use Belleville Airport are considering buying Skybuses.
(E) The aerodynamic design of the Skybus causes turbulence behind it when taking off that forces other planes on the runway to delay their takeoffs.


4.    (24257-!-item-!-188;#058&001200)


Which of the following most logically completes the argument?


A new machine for harvesting corn will allow rows to be planted only fifteen inches apart, instead of the usual thirty inches.  Corn planted this closely will produce lower yields per plant.  Nevertheless, the new machine will allow corn growers to double their profits per acre because __________.


(A) with the closer spacing of the rows, the growing corn plants will quickly form a dense canopy of leaves, which will, by shading the ground, minimize the need for costly weed control and irrigation
(B) with the closer spacing of the rows, corn plants will be forced to grow taller because of increased competition for sunlight from neighboring corn plants
(C) with the larger number of plants growing per acre, more fertilizer will be required
(D) with the spacing between rows cut by half, the number of plants grown per acre will almost double
(E) with the closer spacing of the rows, the acreage on which corn is planted will be utilized much more intensively than it was before, requiring more frequent fallow years in which corn fields are left unplanted

作者: balapupu    时间: 2011-12-7 21:36
Question #6. Weaken-CE. December 2000 LSAT, Section 2, #7. The correct answer choice is (A)
The heart of the argument is a causal claim that the reason today’s newspapers are full of stories about
violent crime is that violent crime has now risen to a point where it is out of control.
MVC = more violent crimes
NFS = more news stories about violent crimes
C E
MVC NFS
On the basis of this relationship, the author adds for good measure that one should not leave one’s home!
From a form standpoint, this problem is very similar to #3 in this problem set. This is one of the keys to the
LSAT—you must recognize the patterns that exist within the test and then capitalize on them when they
appear. All the problems in the set include causality. From this point on, you must recognize causality
when it appears and then properly respond to it. Your ability to recognize these forms will give you an
advantage in both speed and confidence, and ultimately raise your score.
Answer choice (A): This is the correct answer. This answer presents an alternate cause to the scenario
presented above, namely that more comprehensive coverage leads to more news stories, not more violent
Answer choice (B): This Opposite answer strengthens the argument.
Answer choice (C): This answer strengthens the argument, if anything.
Answer choice (D): This is an answer that many people select. The answer is incorrect because it fails to
account for other violent crimes beside murder. Indicating that murder comprised a higher percentage of
violent crimes in the old days than today does not address the total number of crimes being committed.
Since the argument concludes that “violent crime is now out of control,” this answer is incorrect. Consider
the following example:
Old Days Today
Total number of murders 3 1,000
Total number of violent crimes 4 50,000
Percentage of violent crimes
that are murders 75% 2%
In this example, although murder was a higher proportion of the violent crimes in the old days, today there
are many more violent crimes. This shows that the scenario in the answer choice does not have to
undermine the argument. In the chapter on Numbers and Percentages we will revisit the concept of
proportion versus total numbers and discuss how the test makers use numerical ideas to attack test takers.
Answer choice (E): The role played by magazines in informing the public does not address why there are
so many stories about violent crime or if violent crime is now out of control.

逻辑链: 1. D   2. C  3. E   4. A
作者: xeyyxzty    时间: 2011-12-7 21:58
1.premise:Unlike newspapers in the old days, today’s newspapers and televised news programs are full of stories about murders and assaults in our city. conclusion:violent crime is now out of control, and, to be safe from personal attack, one should not leave one’s home except for absolute necessities.
newspapers report more about crimes today
(A) Newspapers and televised news programs have more comprehensive coverage of violent crime than newspapers did in the old days.
--right
(B) National data show that violent crime is out of control everywhere, not just in the author’s city.
--support
(C) Police records show that people experience more violent crimes in their own neighborhoods than they do outside their neighborhoods.
--irrelevant
(D) Murder comprised a larger proportion of violent crimes in the old days than it does today.
--irrelevant
(E) News magazines play a more important role today in informing the public about crime than they did in the old days.
--irrelevant
you must recognize the patterns that exist within the test and then capitalize on them when they appear. All the problems in the set include causality. From this point on, you must recognize causality when it appears and then properly respond to it. Your ability to recognize these forms will give you an advantage in both speed and confidence, and ultimately raise your score.
2.On Pacific islands, a newly arrived gecko species, the house gecko, is displacing the previously established mourning gecko in urban areas, but populations of the two species are more stable in rural areas far from human settlement.  The house gecko does not attack the mourning gecko, but in areas where insects congregate it prevents the mourning gecko from feeding on them.
Which of the following contributes most to an explanation of the difference between gecko populations in urban and rural areas?there are no insects in urban areas?
(A) In urban areas, geckos are valued because they eat so many insects.
--irrelevant
(B) Geckos defend territories against other members of the same species.--irrelevant
(C) House geckos that arrive on islands are carried there in boats and planes.
--irrelevant
(D) In places where there are lighted buildings, insects tend to gather around the light.
--right
(E) Mourning geckos are all females and reproduce asexually, but house geckos reproduce sexually.
--irrelevant
3.background information:Downtown Villieu was once a flourishing business district, but most Villieu-area businesses are now located only in the suburbs.  The office buildings downtown lack the modern amenities most business operators demand today.  premise: Villieu officials plan to have several modern office buildings constructed and to offer reduced local tax rates to any business that leases space in a new downtown building.
conclusion:lure more businesses downtown
(A) Most of the businesses currently located in downtown Villieu have long-term leases on the space they occupy there.
--irrelevant
(B) The existing office buildings in downtown Villieu have, on average, a much higher vacancy rate than do office buildings in Villieu's suburbs.
--irrelevant
(C) The local tax rates in Villieu's suburbs are significantly lower than downtown Villieu's proposed rate for businesses that lease space in the new office buildings.
--right
(D) Most of the businesses that currently lease office space in downtown Villieu also lease office space in Villieu's suburbs.
--irrelevant
(E) With the new office buildings, downtown Villieu would have a greater amount of modern office space than any other downtown business district in the region.
--irrelevant
4. background information:Northern Air has dozens of flights daily into and out of Belleville Airport, which is highly congested.  Northern Air depends for its success on economy and quick turn around and consequently is planning to replace its large planes with Skybuses, whose novel aerodynamic design is extremely fuel efficient.
premise:The Skybus' fuel efficiency results in both lower fuel costs and reduced time spent refueling.
conclusion:ease the problem
(A) The Skybus would enable Northern Air to schedule direct flights to destinations that currently require stops for refueling.
--support
(B) Aviation fuel is projected to decline in price over the next several years.
--support
(C) The fuel efficiency of the Skybus would enable Northern Air to eliminate refueling at some of its destinations, but several mechanics would lose their jobs.
--irrelevant, or support
(D) None of Northern Air's competitors that use Belleville Airport are considering buying Skybuses.
--irrelevant
(E) The aerodynamic design of the Skybus causes turbulence behind it when taking off that forces other planes on the runway to delay their takeoffs.
--right
5.
background information:A new machine for harvesting corn will allow rows to be planted only fifteen inches apart, instead of the usual thirty inches.  
premise:Corn planted this closely will produce lower yields per plant.  + ???
conclusion: the new machine will allow corn growers to double their profits per acre
less loss by insects?
(A) with the closer spacing of the rows, the growing corn plants will quickly form a dense canopy of leaves, which will, by shading the ground, minimize the need for costly weed control and irrigation
--right
(B) with the closer spacing of the rows, corn plants will be forced to grow taller because of increased competition for sunlight from neighboring corn plants
(C) with the larger number of plants growing per acre, more fertilizer will be required

--opposite to the information, or irrelevant
(D) with the spacing between rows cut by half, the number of plants grown per acre will almost double

--so harvest less than 2 times
(E) with the closer spacing of the rows, the acreage on which corn is planted will be utilized much more intensively than it was before, requiring more frequent fallow years in which corn fields are left unplanted

--irrelevant
作者: fox0923    时间: 2011-12-7 23:56
精练-----------------------26s------------------------------weaken
P: Unlike the old newspaper, the current newspaper reports more violent crimes.
C: Therefore, people should not leave their houses since more and more crimes occur.
Prephrase: The old newspaper didn't report crimes as many as current newspaper.


Analysis:
(A) Newspapers and televised news programs have

more comprehensive coverage of violent
crime than newspapers did in the old days.------------------------------this is correct. IF the coverage of violent crime was not as comprehensive as it is now, certainly the report of crimes of old newspaper would be less.
(B) National data show that violent crime is out of
control everywhere, not just in the author’s
city.------------------------------------------------------------------------------support the argument.
(C) Police records show that people experience
more violent crimes in their own
neighborhoods than they do outside their
neighborhoods.-----------------------------------------------------------------where the violent crimes happen is not an issue
(D) Murder comprised a larger proportion of
violent crimes in the old days than it does
today.----------------------------------------------------------------------------a large proportion of violent crimes doesn't mean large number of crimes.
(E) News magazines play a more important role
today in informing the public about crime
than they did in the old days.------------------------------------------------this can be a contender at first, but then it cannot conclude the reason that the news magazine report more crimes than the old magazines did.

作者: zz42050524    时间: 2011-12-8 09:05
Weaken
Newspapers of today are full of stories about violent crimes. It refers that the society is out of control.
People should not leave home unless absolutely necessary.
Pre: 1. Newspapers are more likely to print stories about violent crimes, therefore it looks like the society is dangerous but indeed it is not.
A) Newspapers and televised news programs have  contender
more comprehensive coverage of violent
crime than newspapers did in the old days.  
(B) National data show that violent crime is out of  support
control everywhere, not just in the author’s
city.
(C) Police records show that people experience  irrelevant
more violent crimes in their own
neighborhoods than they do outside their
neighborhoods.
(D) Murder comprised a larger proportion of    irrelevant
violent crimes in the old days than it does
today.
(E) News magazines play a more important role    contender
today in informing the public about crime
than they did in the old days.

这道题  A E让我犹豫,仔细想了下,还是A对, E是一个无关项。 A突出了 more coverage ,解释了是newspaper加大了范围而不是社会out of control,weaken了论述。
作者: qiuhua01234567    时间: 2011-12-8 11:56

作者: winghyy    时间: 2011-12-8 12:57
精炼题
25s-weaken
Today's newspaper and TV news are full of violent stories.
One can conclude that violent crime is now out of control and one should not leave home except for absolute necessities.
Prephase: Violent stories are now allowed to be published on newspapers and TV. They are prohibitted in the old days.
选A
(A) Newspapers and televised news programs have more comprehensive coverage of violent
crime than newspapers did in the old days.——correct
(B) National data show that violent crime is out of control everywhere, not just in the author’s
city. ——Strengthen
(C) Police records show that people experience more violent crimes in their own
neighborhoods than they do outside their neighborhoods.—— ——Strengthen
(D) Murder comprised a larger proportion of
violent crimes in the old days than it does
today.——a large proportion doesn't mean the large number of crimes
(E) News magazines play a more important role
today in informing the public about crime——irrelevant(play a role)
than they did in the old days.
作者: balapupu    时间: 2011-12-9 08:33
1.【19s】
P:
Today the crime and violent stories in the newspapers are more than before? the crimes and violent issues are out of control.
C: people should stay at home.
Weaken: newspaper always describes the thing does not happen in the real life.
A:
(A) Newspapers and televised news programs have
More comprehensive coverage of violent
Crime than newspapers did in the old days.-->R
(B) National data show that violent crime is out of
Control everywhere, not just in the author’s
city.-->irrelevant with other places.
(C) Police records show that people experience
More violent crimes in their own
Neighborhoods than they do outside their
neighborhoods. -->irrelevant comparison.
(D) Murder comprised a larger proportion of
Violent crimes in the old days than it does
today.-->irrelevant comparison.
(E) News magazines play a more important role
today in informing the public about crime
than they did in the old days.-->support
作者: balapupu    时间: 2011-12-9 08:52
逻辑链第四题做错了额。。。。。现在想想是自己粗心了~~

(A) with the closer spacing of the rows, the growing corn plants will quickly form a dense canopy of leaves, which will, by shading the ground, minimize the need for costly weed control and irrigation---->possible
(B) with the closer spacing of the rows, corn plants will be forced to grow taller because of increased competition for sunlight from neighboring corn plants-->grow taller does not mean they produce more.
(C) with the larger number of plants growing per acre, more fertilizer will be required-->weaken
(D) with the spacing between rows cut by half, the number of plants grown per acre will almost double-->tricky!!! the number may double, however,  the passage already mention that there will be less yields, so the total production will less than double the amount.
(E) with the closer spacing of the rows, the acreage on which corn is planted will be utilized much more intensively than it was before, requiring more frequent fallow years in which corn fields are left unplanted->weaken
作者: 风无衣    时间: 2011-12-9 16:07
【精炼1-19】
weaken_39s
premise:today newspapers and TV news programs report more about murders and assaults than newspapers in the past.
conclusion:the violient crime is out of control and for one's safety,one should not leave one alone at home unless for absolute  necessities.
(A) Newspapers and televised news programs have
more comprehensive coverage of violent
crime than newspapers did in the old days.——correct
(B) National data show that violent crime is out of
control everywhere, not just in the author’s
city.——has no effect on this issue
(C) Police records show that people experience
more violent crimes in their own
neighborhoods than they do outside their
neighborhoods.——support
(D) Murder comprised a larger proportion of
violent crimes in the old days than it does
today.——has no effect on this issue
(E) News magazines play a more important role
today in informing the public about crime
than they did in the old days.——it is about the newspapers or news programs.has nothing to do with the news magazines.

虽然选对咯~贴个D项的解释再看看~嘻嘻~
Answer choice (D): This is an answer that many people select. The answer is incorrect because it fails to
account for other violent crimes beside murder.
Indicating that murder comprised a higher percentage of
violent crimes in the old days than today does not address the total number of crimes being committed.
Since the argument concludes that “violent crime is now out of control,” this answer is incorrect. Consider
the following example:
Old Days Today
Total number of murders 3 1,000
Total number of violent crimes 4 50,000
Percentage of violent crimes
that are murders 75% 2%
In this example, although murder was a higher proportion of the violent crimes in the old days, today there
are many more violent crimes. This shows that the scenario in the answer choice does not have to
undermine the argument. In the chapter on Numbers and Percentages we will revisit the concept of
proportion versus total numbers and discuss how the test makers use numerical ideas to attack test takers.
percentage和number~相对数和绝对数~attention!
作者: ugly5552000    时间: 2011-12-13 00:51
1/Background: today’s newspaper and televised news program are full of stories about murders and assaults in the city.
Premise: the violent crime is out of control, for the sake of life security, one should not leave home unless it is absolutely necessary.
Prephrase: according to the report of police, the violent crime has stayed on the same level in the recent ten years.
A
A: the news programs have more comprehensive coverage of violent crime, so more news about violent crimes exist, this does not mean that the violent crimes increase.
B: supports the argument
C: irrelevant and in some extent, this supports the argument.
D: larger percentage does not mean more violent crimes, the concrete amount depends on the radix number.
E: irrelevant
2/Background: the house gecko, a newly gecko species, is displacing the previously established mourning gecko in urban areas. The populations of the two species are more stable in rural areas. The house gecko does not attack the mourning gecko, but prevent the mourning gecko from feeding in areas where insects congregate.
Prephrase: it is in the urban areas where the insects are often congregate. Because of being prevented from feeding on the insects, the mourning gecko is displaced by house gecko.
D
A: irrelevant
B: irrelevant
C: irrelevant
D: where there are more insects, there are more house geckos
E: irrelevant
3/Background: few businesses locate in downtown vilieu, because the office downtown building lack the modern amenities most business operators demand.
Premise: Vilieu officials plan to constructed modern office buildings and to lure more businesses downtown by offering reduced local tax rates to any business that leases space in a new downtown building.
Prephrase: the modern amenities are still inadequate.
C
A: supports the arguments
B: irrelevant
C: since the tax rates in suburbs are much lower, few businesses are willing to lease space downtown.
D: supports the arguments
E: irrelevant
4/Background: Northern Air plans to replace its large planes with Skybus.
Premise: Skybus is more fuel efficient, which results in both lower fuel costs and reduced time spent refueling.
Prephrase: Skybus carries less passengers and covers smaller areas.
A
A: supports the argument: because the skybus does not need the current stops for refueling.
B: supports the argument
C: irrelevant
D: irrelevant
E : the turbulence caused by skybus will have serious influence on other planes
5/Background: a new machine for harvesting corn allows rows to be planted in higher density. Closely planted corn will produce lower yields. However, the machine will allow corn growers to double their profits per acre.
Prephrase: the machine reduces the harvest cost and the increase of yields in whole acre covers the lost of yields per plant.
A
A: Because of the machine, the corns are planted more densely, the need for costly weed control and irrigation is minimized.
B: irrelevant
C: weakens the argument
D: doubled the number of plants does not guarantee the double profit
E : irrelevant
作者: leewonting    时间: 2012-4-17 09:44
97
Unlike newspapers in the old days, today’s newspapers and televised news programs are full of stories about murders and assaults in our city. One can only conclude from this change that violent crime is now out of control, and, to be safe from personal attack, one should not leave one’s home except for absolute necessities. Which one of the following, if true, would cast the most serious doubt on the conclusion?
(A) Newspapers and televised news programs have more comprehensive coverage of violent crime than newspapers did in the old days.
(B) National data show that violent crime is out of control everywhere, not just in the author’s city.
(C) Police records show that people experience more violent crimes in their own neighborhoods than they do outside their neighborhoods.
(D) Murder comprised a larger proportion of violent crimes in the old days than it does today.
(E) News magazines play a more important role today in informing the public about crime than they did in the old days.
背景:Today’s newspaper and televised programs are full of stories about murders and assaults in the city.
条件:One can be explained that violent crime is now out of control.
结论:One should stay home unless absolute necessities
推测:未必是犯罪的上升,或者是对这个更开放;不出门也太不靠谱了吧,就算是这么解释,也有出门也安全的方法(或者小偷、凶手入室的更多?)
选C
答案应该选A,comprehensive coverage是全面覆盖,(if anything,如果有的话),A比C更有说服力。


98
背景:On Pacific island, a newly arrived gecko species, the horse gecko, is displacing the previously established mourning gecko in urban areas.
条件:The populations of the two species are more stable in rural areas far from human settlement
结论:The horse gecko does not attack the mourning gecko, but in areas where insects congregate it prevents the mourning gecko from feeding on them.
推测:insects不充足的时候才会attack,人多了,环境污染了……
选D


99
背景:Downtown Villieu was once a flourishing business district, but most Villieu-area businesses are now located only in the suburbs.
条件:Office buildings downtown lack the modern amenities most business operators demand today. So construct the buildings and reduce the local tax rates will lure more businesses
结论:The plan will succeed
推测:除了建筑和税的问题,suburban有比downtown更吸引的地方
选C

100
背景:Northern Air has dozens of flights daily into and out of Belleville Airport, which is highly congested.
条件:Northern Air succeeds because it can quickly turn around. And it wants to replace its large plane with Skybus that is extremely fuel efficient
结论:Result in lower cost and reduce time spending
推测:那么大的灰机,会不会很难turn around啊?
选E


101
Which of the following most logically completes the argument?
A new machine for harvesting corn will allow rows to be planted only fifteen inches apart, instead of the usual thirty inches.  Corn planted this closely will produce lower yields per plant.  Nevertheless, the new machine will allow corn growers to double their profits per acre because __________.
(A) with the closer spacing of the rows, the growing corn plants will quickly form a dense canopy of leaves, which will, by shading the ground, minimize the need for costly weed control and irrigation
(B) with the closer spacing of the rows, corn plants will be forced to grow taller because of increased competition for sunlight from neighboring corn plants
(C) with the larger number of plants growing per acre, more fertilizer will be required
(D) with the spacing between rows cut by half, the number of plants grown per acre will almost double
(E) with the closer spacing of the rows, the acreage on which corn is planted will be utilized much more intensively than it was before, requiring more frequent fallow years in which corn fields are left unplanted
背景:Corn usually is planted thirty inches apart, and there is a new machine for harvesting corn will allow rows to be fifteen inches apart
条件:Corn planted this closely will produce lower yields per plant
结论:But corn growers will double their profits because of using the new machine
推测:虽然per plant的产量减少,但是他们种了很多plant
选D
答案应该选A,虽然每plant的产量减少了,但是同时用于灌溉和杂草控制的药物也减少了,弥补了每plant减少而导致的钱的减少。
D,并没有increase profit,只是保持了!!!!!!!!!!!!!!!!!!!!
作者: Roxas605    时间: 2012-4-25 15:06
2.    
Downtown Villieu was once a flourishing business district, but most Villieu-area businesses are now located only in the suburbs.  The office buildings downtown lack the modern amenities most business operators demand today.  To lure more businesses downtown, Villieu officials plan to have several modern office buildings constructed and to offer reduced local tax rates to any business that leases space in a new downtown building.

Which of the following, if true, most threatens the plan's likelihood of success?

(A) Most of the businesses currently located in downtown Villieu have long-term leases on the space they occupy there.
(B) The existing office buildings in downtown Villieu have, on average, a much higher vacancy rate than do office buildings in Villieu's suburbs.
(C) The local tax rates in Villieu's suburbs are significantly lower than downtown Villieu's proposed rate for businesses that lease space in the new office buildings. 【proposed rate=rates have reduced!!!之前误以为是调整前,但如果是lease space的一定是政策后了】


(D) Most of the businesses that currently lease office space in downtown Villieu also lease office space in Villieu's suburbs.
(E) With the new office buildings, downtown Villieu would have a greater amount of modern office space than any other downtown business district in the region.


【前提】:Building(downtown)需要修理---->business不在downtown
【结论】:(1)建设modern building
                 (2)减免税率                         ------------->more lease(租)
【预测】:  weaken:   1.税率减免之后还是很高
                                2.新的modern building还是不如suburbs


PS:我是这样理解A的,因为是long term合同,所以不能退掉现在的lease,而去租新的building,所以条件不成立,计划无效。
WHY????
作者: FB小贝    时间: 2012-5-15 22:32
6. Unlike newspapers in the old days, today’s
newspapers and televised news programs are full of
stories about murders and assaults in our city. One
can only conclude from this change that violent
crime is now out of control, and, to be safe from
personal attack, one should not leave one’s home
except for absolute necessities.
Which one of the following, if true, would cast the
most serious doubt on the conclusion?
(A) Newspapers and televised news programs have
more comprehensive coverage of violent
crime than newspapers did in the old days.
(B) National data show that violent crime is out of
control everywhere, not just in the author’s
city.
(C) Police records show that people experience
more violent crimes in their own
neighborhoods than they do outside their
neighborhoods.
(D) Murder comprised a larger proportion of
violent crimes in the old days than it does
today.
(E) News magazines play a more important role
today in informing the public about crime
than they did in the old days.

A
作者: spencerX    时间: 2012-5-30 21:13
【精练】
Premise: Today’s newspapers and tv programs are full of stories about murder and assaults.
Conclusion:
1.    One can only conclude than violent crime is out of control now.
2.    To avoid the crime, we should not leave our home
Pre: There is a difference between the actually violent crimes and what the news papers or tv programs reported.
Analysis:
A.    Correct. This statement can explain why news papers and TV programs report more violent crimes nowadays—not because there are more violent crimes in our society. So the argument made by the author is weakened.
B.    Opposite answer. This statement actually support the argument by giving the evidence that there are more violent crimes everywhere.
C.    Irrelevant
D.    Irrelevant
E.    Irrelevant. What concerned in the argument is news papers, not news magazines

【逻辑链】
1.  
Fact: H g is displacing M g in urban area, the population of the two species are more stable in rural areas far from human settlement. H doesn’t attack M, in areas where insects congregate: m feed less on them
Pre: In the human settlement, there are more insects.
My answer: D

2.
Background: Most V—area business are now located only in the suburbs. the office buildings cannot meet the requirements of the business.
Premise:
1.    build several modern office buildings
2.    reduce local tax rates
Conclusion: Lure more business
Pre:
1.    The building constructed by the government still cannot meet the standard of V—area business
2.    The tax rates is still not low enough
My answer: C

3.
Background: Northern Air use B Airport, which is very congested.
Premise: Northern Air depends for its success on efficiency, so they decided to replace their planes with Skybuses, because it is fuel efficient
Conclusion: The Skybus' fuel efficiency results in both lower fuel costs and reduced time spent refueling.
Pre: Other facts may cause much more time needed to turn around, and thus can reduce the efficiency of NA
My answer: E

4.
Background: A new machine for harvesting corn will allow rows to be planted only fifteen inches apart, instead of the usual thirty inches
Premise: Corn planted this closely will produce lower yields per plant.
Conclusion: the new machine will allow corn growers to double their profits per acre
Pre: The areas that can grow corn increase.
My answer: D
错误原因:选项D只能保证收入不变,但是不能得出grows将double他们的profits
而选项B则直接表明成本可以降低,那么grows将可能double profits per acre
作者: baseboss    时间: 2012-6-1 20:33
1.A lot of news about voilent crime.The verdict about this phenomenon is voilent crime surrand us.People without necessary should not go out.
presume:
Report more ,do not means occurring more.

(A) Newspapers and televised news programs have
more comprehensive coverage of violent
crime than newspapers did in the old days.
-->correct
(B) National data show that violent crime is out of
control everywhere, not just in the author’s
city.
-->support
(C) Police records show that people experience
more violent crimes in their own
neighborhoods than they do outside their
neighborhoods.
-->irrelevant
(D) Murder comprised a larger proportion of
violent crimes in the old days than it does
today.
-->irrelevant
(E) News magazines play a more important role
today in informing the public about crime
than they did in the old days.
-->irrelevant

2 G are more stable in rural far from the human  settlement.
HG will prevent MG feeding on the insect.
presume:
insect congregated in the light area.
D

3
Presume:
modern building construction and low tax can not meet the company's demand.
C

4
Presume:
Equipment for current planes and for the skybus' will be the same or not.
E

5
Presume:
Machine can reduece the harvest time cost ,this can be implement the yield reduce.
A
作者: 二楼往下掉    时间: 2012-6-16 00:15
1.
(1)25s weaken
(2)逻辑链
Primes:
Today’s newspapers and televised news programs are full of stories about murders and assaults in our city
Conclusion:
Violent crime is now out of control and one should not leave one’s home except for absolute necessities
(3)<1>现在比过去传媒要先进很多,所以即使报道的谋杀案多了,也不能推出现在暴力犯罪已经失去控制
<2>统计表明,大比例谋杀案发生在家里
(4)A:现在的报纸电视比原来的报纸对于暴力犯罪的覆盖面更全面了,没有对conclusion造成影响,loser
B:这应该算是support,依然承认在作者的城市暴力犯罪失去控制,loser
C:同预测<2>,contender
D:谋杀占暴力犯罪的比例比现在大,与结论无关,loser
E:magazine在文中完全没提到,loser
所以,选C
正确答案:A

2.
(1)1’ explanation
(2)逻辑链
Fact:
<1>On Pacific islands, a newly arrived gecko species, the house gecko, is displacing the previously established mourning gecko in urban areas, but populations of the two species are more stable in rural areas far from human settlement.
<2> The house gecko does not attack the mourning gecko, but in areas where insects congregate it prevents the mourning gecko from feeding on them.
(4)A:没有解释为什么在城市和乡村两种壁虎的数量比例差很多,loser
B:壁虎攻击同类,没有解释问题,loser
C:坐飞机和船来又怎样。。。,loser
D:昆虫喜欢去亮的地方扎堆,house gecko不许mourning gecko捕食昆虫,所以mourning就不在urban待着了?
E:与生殖方式无关,loser

3.
(1)30s weaken
(2)逻辑链
Background:
<1>Downtown Villieu was once a flourishing business district, but most Villieu-area businesses are now located only in the suburbs.
<2>The office buildings downtown lack the modern amenities most business operators demand today.
Primes;
Villieu officials plan to have several modern office buildings constructed and to offer reduced local tax rates to any business that leases space in a new downtown building
Conclusion:
To lure more businesses downtown
(3)<1>搬家成本高于免的税,不划算
<2>几栋现代的办公楼不成规模,不利于做生意
(4)A:文章讨论的重点是不在市中心的这些企业,而不是已经在市中心的这些企业,loser
B:不需要比较现在在市中心的这些楼,应该要比较市中心要新建的楼和郊外的楼,loser
C:还是郊外划算,同<1>,contender
D:在市中心有楼的在郊外也有楼,但是在市中心已经有楼的不在讨论范围,loser
E:说明市中心很好,很有竞争力,所以计划会成功,support,loser
所以,选C

4.
(1)38s weaken
(2)逻辑链
Background:
Northern Air has dozens of flights daily into and out of Belleville Airport, which is highly congested.
Primes:
<1>novel aerodynamic design is extremely fuel efficient.
<2> The Skybus' fuel efficiency results in both lower fuel costs and reduced time spent refueling.
Conclusion:
Northern Air depends for its success on economy and quick turnaround and consequently is planning to replace its large planes with Skybuses
(3)小飞机每一架的成本低,但是装的人少,需要飞更多次,所以总成本比原来高
(4)A:换飞机好啊,没提到换飞机的缺点,loser
B:飞机油在未来几年要降价,所以可以再某种程度上打击结论,contender
C:换飞机可以不需要中途补给了,好事,虽然有几台机器没工作了,但是总的来说是好事,loser
D:别人家使用B机场都没有考虑换S飞机,说明S飞机有可能有点问题?contender
E:三角架靠后,需要别的飞机退后起飞,这不地道,contender
所以,尽管B,D都在某种程度上伤害到了conclusion,但是没有明确点出S飞机到底哪里有问题,E明确说出S会导致别的飞机有问题,所以不能实现NAa公司的目标,选E

5.
(1)40s explain
(2)逻辑链
Background:
<1>A new machine for harvesting corn will allow rows to be planted only fifteen inches apart, instead of the usual thirty inches.
<2>Corn planted this closely will produce lower yields per plant.
Primes:
?
Conclusion:
The new machine will allow corn growers to double their profits per acre
(3)机器收割的快可以一年种两回
(4)A:种的密了叶子长出来会遮挡阳光,这样省了除杂草的钱,contender
B:种密了会长的更高,但是没有说收成如何,不能把长得高=收成好,loser
C:种密了需要更多的肥料,费钱啊,loser
D:这跟文中的背景冲突,loser
E:种密了需要更长时间的休耕,应该不是赚钱的买卖,loser
所以,选A
作者: jetyxo    时间: 2012-6-21 19:59
1 22"  premise: today, paper and televison brodcast more about crimes than before
         conclusion: crimes are out of control
         weaken?  today, brodcasting more like to  focus on crimes than before---->A
----------------------------------------------------------------------
2 30"  background: in rural areas, HG and MG ' population keep stable
                              when in area where insects concentrated, HG precents MG from feeding on the insect
         reason? why the population of both HG and MG is different in city
                       in city ,there have more insects concentrated than in rural areas---->D
--------------------------------------------------------------------
3 31"  premise: in order to attract more company to lease in downtown, goverment plan to build more moden building and reduce  tax rate
         weaken? thsi plan will fail to lure company
          answer : C    the plan tax rate is still not attractive than that of  suburb
------------------------------------------------------------
4 25" background:  NA is success in its economic and quick turnover
        premise: NA  repalce its airpalnes with a  kind of new one ,which is more fuel efficient and can reduce time needed to fuel
        weaken? the new airplane may be has other bad effect on economic or time consum--->E
        A B C support D irrelevant
---------------------------------------------------------------
5 23" background: a new technology can increase the number of corn planted in per acre, but will reduce the production
        conclusion: the profit will double
        reason? may be the technology will take other advantage to set off the reduce in production   --->A
作者: emmasy    时间: 2012-7-8 23:30
1.
facts:newspapers and televised news programs are full of
stories about murders and assaults
conclusion:violent crime is now out of control, and, to be safe from personal attack, one should not leave one’s home except for absolute necessities.
a.support
b.irrevelant
c.irrevelant
d.irrevelant
e.weaken

2.
d.correct

3.
aim:To lure more businesses downtown
solusion:have several modern office buildings constructed and to offer reduced local tax rates to any business that leases space in a new downtown building
a.irrevelant
b.irrevelant
c.weaken
d.irrevelant
e.irrevelant

4.
aim: economy and quick turnaround
solusion:replace its large planes with Skybuses, whose fuel efficiency results in both lower fuel costs and reduced time spent refueling.
a.support
b.irrevelant
c.irrevelant
d.irrevelant
e.weaken

5.
a.correct
b.irrevelant
c.weaken
d.irrevelant
e.weaken
作者: LuckyYolandaLi    时间: 2012-7-11 22:09
6.
1)计时:
2)逻辑链:
Premise: Unlike newspapers in the old days, today’s newspapers and televised news programs are full of stories about murders and assaults in our city.
Conclusion: violent crime is now out of control, and, to be safe from personal attack, one should not leave one’s home except for absolute necessities.
3)推测:Today’s newspapers and televised news programs are full of stories about murders and assaults in our city in order to attract more attention.
4)选项分析:选A
A: Correct.
B: Irrelevant (or strengthen).
C: Irrelevant.
D: Irrelevant.
E: News magazines——irrelevant.


1.
1)计时:s
2)逻辑链:
Premise: The house gecko does not attack the mourning gecko, but in areas where insects congregate it prevents the mourning gecko from feeding on them.
Conclusion: On Pacific islands, a newly arrived gecko species, the house gecko, is displacing the previously established mourning gecko in urban areas, but populations of the two species are more stable in rural areas far from human settlement.
3)推测:Insects congregate in urban areas.
4)选项分析:选D
A: Irrelevant.
B: Irrelevant.
C: Irrelevant.
D: Correct.
E: Irrelevant.


2.
1)计时:s
2)逻辑链:
Premise: Downtown Villieu was once a flourishing business district, but most Villieu-area businesses are now located only in the suburbs.  The office buildings downtown lack the modern amenities most business operators demand today.
Conclusion: To lure more businesses downtown, Villieu officials plan to have several modern office buildings constructed and to offer reduced local tax rates to any business that leases space in a new downtown building.
3)推测:。。。
4)选项分析:选B(正确答案:C
A: Irrelevant.
B: Correct.
C: 这个刚开始也觉的有点像。。。
D: Irrelevant.
E: Irrelevant.
还是不太明白。。。



3.
1)计时:s
2)逻辑链:
Premise: Northern Air has dozens of flights daily into and out of Belleville Airport
, which is highly congested.  Northern Air depends for its success on economy and quick turnaround and consequently is planning to replace its large planes with Skybuses, whose novel aerodynamic design is extremely fuel efficient.Conclusion: The Skybus' fuel efficiency results in both lower fuel costs and reduced time spent refueling.
3)推测:Skybus can hold less passengers.
4)选项分析:选E
A: Counteractive.
B: Irrelevant.
C: 我觉得这个也有点像
D: Irrelevant.
E:Correct.


4.
1)计时:s
2)逻辑链:
Premise: A new machine for harvesting corn will allow rows to be planted only fifteen inches apart, instead of the usual thirty inches.  Corn planted this closely will produce lower yields per plant.
Conclusion: The new machine will allow corn growers to double their profits per acre
3)推测:Much more corn can be planted in the areas with the same size
4)选项分析:选A
A: Correct.
B: Irrelevant.
C: Counteractive.
D: 这题真心纠结了,只能认为这种情况即使增长也达不到二倍
E: Counteractive.
作者: cleotina    时间: 2012-7-15 12:43
7-14 (凡事都有因果, 弄清因果关系)
40''
weaken
1, more crimes and murders are reported in nowadays Newpaper
C: 1 violent crime is now out of control
  2 to be safe should stay at home
Guess: 1 in the old days, there were more crime, but the mass media not report
      2 most of the crimes are commita at resident's home
OA: A
C: suspect
D: strong


1'52''(着实不明白,好多不确定的单词)
explain the dispency
1 HG were displaced in the place where MG established
2 HG and MG are more stable in the rural than in urban place
3 HG did not atack the MG, but in the insects congregate place it can prove MG feed on them(the insects)
guess: if gecko lack insects to eat, they tend to fight for a large territory.
OA: D


36''
weaken
1. more business in the suburben, not in downtown
2, so, there are no facilities to support the business in the downtowm
3, In order to improve the business in the downtown
4, plan to build two business building in the dwontowm with facilities to lease
Guess: the reason business tend to move out of downtown is others factors . not the pionts that they plan to improve
OA: C
E: strong
Others: Irrelevant

1'06''
weaken
1 NA has dozens of flights daily in the airport, which is a little congestion.
2 NA make profits depend for economic and quick turnaround
3 they plan to change into new plains, which is feul efficiency and reduce the time spend refuling.
Guess: the more money spend is on other time and other terms
OA: C
E: suspect, However, it need more infer to get a conclusion
错: 自己的问题引发的后果要自己负责任, 不可以拍屁股走人
30''
1 a kind of machine can allow the pant row every 15 inches but not as old day 30 inchers per row.
2 plant so closely may lead to a low production of each plant
3 use this mathcine can double their profits
Guess: they can plant more than doble the plants
OA
错: 经近两倍但仍旧不是两倍  要严谨
作者: Donts    时间: 2012-7-18 11:14
精练题 计时31''premise:unlike newspapers in old days, the newspaper and televised news program are full of violent news today.
conclusion:crime now is out of control, one had better not leave home except for absolute necessities.
推测:现在报道过去不报道不一定说明crime out of control
(A) Newspapers and televised news programs have more comprehensive coverage of violent
crime than newspapers did in the old days.覆盖面更广,所以报道会更多。正确。
(B) National data show that violent crime is out of control everywhere, not just in the author’s
city.有加强的意味。
(C) Police records show that people experience more violent crimes in their own neighborhoods than they do outside their neighborhoods.无关。
(D) Murder comprised a larger proportion of violent crimes in the old days than it does today.murder和violent crime有差。
(E) News magazines play a more important role today in informing the public about crime than they did in the old days.无关。
选择A
逻辑链
1.计时44''
premise:the new does not attack the old one, but in areas where the insects congregate

conclude:the new species is displacing the old one but the pop. of the two species are more stable in rural areas far from human settlement.
推测:insects congregate more in Human settlement.
选择D
2.计时43''
background:downtown once was flourishing but now the businesses are in suburbs because office buildings downtown are lack of A businesses need.
goal:to attract business downtown
plan:increase A and reduce tax
推测:suburbs也有A和税收优惠措施,或者business prefers产业集群效应than这两个措施。
选择C
3.计时33''
background:B's airport is congested
plan:replace its large planes with skybuses whose X design is efficient for  its lower fuel costs and reduced time spent refueling.
goal:depends for its success on economy and quick turn around.
选择E
4.计时29''
premise:new machine makes corns to grow more closely so that it will lower the yields per plant.
conclusion:double farmers' profits per arce.
选择D订正:4.A


作者: TICKCOCK    时间: 2012-8-17 00:29
1.    A
2.    D
3.    C
4.    E
5.    B (A)
作者: wanggang0411    时间: 2012-8-27 15:08
25
Premise: More murder story on the media now than before.
Conclusion: Murder is out of control.
Weaken: other reasons caused the effect that there is more story on the media now than before.
Currently media report the murder in whole country now rather than in a limited area beofre.
A: more comprehensive coverage.

28
Premise: HG is displacing MG in urban area but not in rural area.
Premise: HG does not attack MG.
Premise: The area prevent MG from feeding on insects.
Explaination: An answer addressed the difference between gecko poppulation in rural and urban areas is a correct one.
D: If the congregation of insects prevent M gekco from feeding on them, and if the light causes the congregation, this answer explain the issue.

33
Premise: lots of business are now moved to suburbs.
Premise: Buildings downtown lack the necessary modern amendities.
Conclusion: Government plans to build more mordern buildings and reduce the tax rate to any business that leases space here.
Weaken: The tax rates or lease fees in suburbs is lower than that in urban.
C

31
Premise: Airport is congested.
Premise: NA's success depends on it's economy and quick turnaround.
Conclusion: New airplaines have an advantage of fuel efficiency
Weaken: New airplaines cause the airport more congested, and consequently hurt the economy of NA.
E: the turbulence causes the delay of other airplane that hurts the economy of NA.

28
Premise: New machines plant corn fifteen inches apart, but old machines plant corn thirty inches apart.
Premise: The new machines will allow corn growers double their profit
Because: Some other advantages that the old machine doesn't have.
A: lower the cost of weed control and irrigation
作者: 我心匪席    时间: 2012-9-11 11:46
Time: 24”
Premise: Today’s newspaper and televised news programs are full of stories about murders and assaults.
Conclusion: Violent crime is now out of control, and one should stay at home for safety.
Prephase: Today’s readers are more interested in murder’s story.
A----- right weaken
B-----irrelevant
C-----support
D-----irrelevant
E-----irrelevant
=============================================
Time:31”
Background Information: On pacific islands, house gecko displaced mourning gecko in urban areas, but the populations of two species are stable in rural areas.
Premise: The house gecko does not attack the mourning gecko, but in areas where insects congregate HG prevents MG from feeding on them.
Prephase: In urban areas, insects congregate more than those in rural areas.
A-----irrelevant
B-----irrelevant
C-----irrelevant
D-----right
E-----irrelevant
===============================================
Time: 30”
Background Information: Downtown Villieu was once flourishing business district, but most businesses are now located in the suburbs. The buildings downtown lack the modern amenities most business operators demand today.
Premise: Villieu officials plan to have modern office buildings constructed and to offer reduced local tax.
Conclusion: To lure more businesses downtown.
Prephase: Businesses in suburbs are more convenient,
A-----irrelevant
B-----support
C-----right weaken
D-----irrelevant
E-----irrelevant
===========================================
Time: 31”
Background Information: BA is highly congested.
Premise: NA succeeds in its economy and quick turnaround and consequently is planning to replace its large planes with S, whose novel aerodynamic design is extremely fuel efficient.
Conclusion: S’s fuel efficiency results in both lower fuel costs and reduced time spent refueling.
A-----irrelevant
B-----irrelevant
C-----support
D-----irrelevant
E-----right weaken
============================================
Time: 24”
Background Information: New machine will allow corn to be planted only 15 inch apart instead of 30 inch.
Corn will has lower yields per plant
Premise: New machine will allow corn growers to double their profits per acre.
A-----right support
B-----irrelevant
C-----weaken
D-----profit same?
E-----weaken

谢谢楼主~
作者: oytt1111    时间: 2012-10-19 17:28
6
Time: 25s
Premise: today’s newspapers and TV programs are full of stories about murders and assaults
Conclusion: violent crime is now out of control and one should not leave one’s home
Premise: weaken
Perhaps today’s audience like such stories and programs makers just follow audiences’ appetite
Perhaps because such stories can easily catch eyeballs
Perhaps in the old days the newspaper didn’t pay much attention on this subject

Answer: A
A right answer.
B strengthen. It proves author’s conclusion.
C it do weaken the conclusion partly
D opposite the conclusion
E strengthen.

1.
Time: 37s
Conclusion: house is displacing mourning in urban while stable in rural
Premise: House do not attack mourning but prevent mourning from feeding on the insects
Answer: B

2
Time : 72s
Premise: most V business are now located only in suburbs.
Premise: V officials plan to lure more business
Answer: C

3
Time: 33s
Premise: Belleville is congested.
Premise: Northern Air is planning to replace it’s planes with Skybuses.
Premise: Skybus’s efficiency results in both lower fuel costs and time spent refueling.
Answer: E

4
Time: 25s
Premise: new machine allows rows to be fifteen apart, resulting lower yields per plant
Conclusion: new machine allows double profits per acre
Answer: D
作者: ElenW    时间: 2012-10-20 12:23
精练(22.4's)
Bg: unlike newspapers in the old days
pre:newspapers & televised news programs are full of stories about mrders & assaults in our city.
con:violent crime is now out of control .One shoud not leave one's home expect for absolute necessities.
(weaken)
PRE:   C               E
 news report       V. out control & stay home
      C-->E
Ans:A解析说 This answer presents an alternate cause to the scenario
presented above, namely that more comprehensive coverage leads to more news stories, not more violent

B--Opposite answer strengthen
C--strengthen
D--support.解析说The answer is incorrect because it fails to
account for other violent crimes beside murder.
E--irrelevant
逻辑链
1(30.2's)
Bg: populations of the two species are more stable in rural areas far from human settlement.
pre:The house gecko does not attack the mourning gecko
pre: in areas where insects congregate it prevents the mourning gecko from feeding on them.
(resolve the paradox)
PRE: There are another factors to attack the population .Attention: urban& rural areas
Ans--they have exposed under the light and the people will kill them
A--not address the fact
B--irrelevant
C--irrelevant
E--irrelevant
2(30'7s)
Bgowntown Villieu was once a flourishing business district
pre:The office buildings downtown lack the modern amenities most business operators demand today
con:To lure more businesses downtown officials plan to have several modern office buildings constructed and to offer reduced local tax rates to any business that leases space in a new downtown building.
(weaken)
PRE:  C                                  E
new building/ reduced tax rates   lure more business         in fact ,it lacks the modern amenities the business need
Ans: C
A--irrelevant
B--irrelevant
D--no impact
E--irrelevant
作者: srafcatt    时间: 2012-10-21 12:39
精炼 weaken 26s
background:newspaper and TV contain so much violent information
conclusionne should not go out for safety.
my reasoning: most of the violent news criminals have been caught.
choice:E
A if this is right,it is still not safe.so the argument is right
B not convincing
C not convincing
D irrelevant
E correct
----------------------------------------------------------------------
逻辑链
1.explanation 35s
premise:the house gecko don't attack the mouning ones but they have the better ability to feed themselves
conclusion:the house gecko replaced the mourning ones in urban areas but not the rural ones
my explanation: in rural areas insects are abundant
choice
A irrelevant
B irrelevant
C irrelevant
D correct
E irrelevant
2.weaken 36s
downtown V only have business in suburbs,lack amenities,plan to attract business downtown
the method:less tax and new buildings for business
my attempt: building expensive?
choice:A
A correct,weaken
B so what
C strenghten
D irrelevant
E strenghten
3.weaken 27s
problem:congested b airport
solution:economy --replace airplane with time-saving skybuses
my attempt to weakenther aspect to make time a little longer
choice:E
A strenghten
B irrelevant
C irrelevant
D so what
E correct

4.fill 25s
new machine can double the profit
premise: corns planted 15 inches apart and per plant yield declines
my attempt to support:the seeds are cheap and the total amount increase.
choice
A not convincing
B irrelevant
C weaken
D correct
E incorrect information
作者: srafcatt    时间: 2012-10-21 13:37
逻辑链第2题:
农村的office building吸引商户,说郊区的收税要低一点。问加强。
C 收得比现在的优惠政策还要低。正面肯定。正确

A 无关 说downtow 的租户租期时间长。但是这不影响计划最终成功。

4.只有A选项中costly提及了有关profits per acre的东西。
另外D 项说的是同样的面积植株数量增加了将近2倍,不是玉米产量增加将近两倍。

作者: zxppx    时间: 2013-1-16 18:50
6.

Unlike newspapers in the old days, today’s
newspapers and televised news programs are full of
stories about murders and assaults in our city. One
can only conclude from this change that violent
crime is now out of control, and, to be safe from
personal attack, one should not leave one’s home
except for absolute necessities.

The logic chain of the argument is that since more stories about murders and assaults have been reported by newspapers and televised news programs, people should think that violent crime is now out of control and should stay at home avoiding personal attack.
Which one of the following, if true, would cast the
most serious doubt on the conclusion?

(A) Newspapers and televised news programs have
more comprehensive coverage of violent
crime than newspapers did in the old days.

In this case, it is reasonable to find out more violent crimes today than those in old days, but the real threat of violent crimes may not increase. BA
(B) National data show that violent crime is out of
control everywhere, not just in the author’s
city.

Actually, this answer choice strengthens the argument, rather than casts doubt on the conclusion.
(C) Police records show that people experience
more violent crimes in their own
neighborhoods than they do outside their
neighborhoods.

It feels like a Shell Game answer. It seems like the safety in neighborhoods is lower than that is outside neighborhoods. However, the comparison does not weaken the conclusion that people should not leave their homes. For example, if a person leaves his home, there is more likely that a theft would happen in his home; on the other hand, if he stays at home, the probability may be reduced.

Besides, this answer strengthens the argument.
(D) Murder comprised a larger proportion of
violent crimes in the old days than it does
today.

Even though the percentage may decrease, the total number of crimes may increase today.
(E) News magazines play a more important role
today in informing the public about crime
than they did in the old days.

Actually, this answer choice strengthens the argument.



1. (24021-!-item-!-188;#058&000691)
On Pacific islands, a newly arrived gecko species, the house gecko, is displacing the previously established mourning gecko in urban areas, but populations of the two species are more stable in rural areas far from human settlement. The house gecko does not attack the mourning gecko, but in areas where insects congregate it prevents the mourning gecko from feeding on them.
Which of the following contributes most to an explanation of the difference between gecko populations in urban and rural areas?
(A) In urban areas, geckos are valued because they eat so many insects.

This answer choice does not point out the difference between house gecko and mourning gecko.
(B) Geckos defend territories against other members of the same species.

The same to A.
(C) House geckos that arrive on islands are carried there in boats and planes.

The way how house geckos came to islands is outside the scope of the argument.
(D) In places where there are lighted buildings, insects tend to gather around the light.

It seems irrelevant, but a more thorough analysis reveals that it is the best answer. As we know, there are more lighted buildings in urban areas than in rural areas. So it is reasonable to assume that there are more insects congregating in urban areas than in rural areas. What’s more, the argument points out that in areas where insects congregate house gecko prevents mourning gecko from feeding on them. Thus, the logic chain is reasonable. BA
(E) Mourning geckos are all females and reproduce asexually, but house geckos reproduce sexually.
The same to A.



2. (24069-!-item-!-188;#058&000703)
Downtown Villieu was once a flourishing business district, but most Villieu-area businesses are now located only in the suburbs. The office buildings downtown lack the modern amenities most business operators demand today. To lure more businesses downtown, Villieu officials plan to have several modern office buildings constructed and to offer reduced local tax rates to any business that leases space in a new downtown building.
Which of the following, if true, most threatens the plan's likelihood of success?
(A) Most of the businesses currently located in downtown Villieu have long-term leases on the space they occupy there.

It seems to weaken the argument, but if the local tax rates in new buildings are lower than those of their currently located buildings, they may choose the new buildings. So it cannot weaken the argument.
(B) The existing office buildings in downtown Villieu have, on average, a much higher vacancy rate than do office buildings in Villieu's suburbs.

The same to A.
(C) The local tax rates in Villieu's suburbs are significantly lower than downtown Villieu's proposed rate for businesses that lease space in the new office buildings.

In this case, it is reasonable to predict that the plan would not succeed, considering the higher cost to move in new buildings. BA
(D) Most of the businesses that currently lease office space in downtown Villieu also lease office space in Villieu's suburbs.

This answer is irrelevant.
(E) With the new office buildings, downtown Villieu would have a greater amount of modern office space than any other downtown business district in the region.

Actually, this answer choice strengthens the argument.



3. (24117-!-item-!-188;#058&000708)
Northern Air has dozens of flights daily into and out of Belleville Airport, which is highly congested. Northern Air depends for its success on economy and quick turnaround and consequently is planning to replace its large planes with Skybuses, whose novel aerodynamic design is extremely fuel efficient. The Skybus' fuel efficiency results in both lower fuel costs and reduced time spent refueling.
Which of the following, if true, could present the most serious disadvantage for Northern Air in replacing their large planes with Skybuses?
(A) The Skybus would enable Northern Air to schedule direct flights to destinations that currently require stops for refueling.

In fact, this answer choice strengthens the plan.
(B) Aviation fuel is projected to decline in price over the next several years.

It is irrelevant.
(C) The fuel efficiency of the Skybus would enable Northern Air to eliminate refueling at some of its destinations, but several mechanics would lose their jobs.

Unemployment is outside the scope of the argument.
(D) None of Northern Air's competitors that use Belleville Airport are considering buying Skybuses.

Perhaps, they cannot afford the Skybuses, not meaning that this kind of airplane is not good.
(E) The aerodynamic design of the Skybus causes turbulence behind it when taking off that forces other planes on the runway to delay their takeoffs.
In this case, the Skybus would increase delays, thus weakening the advantage of Northern Air, which depends for its success on economy and quick turnaround. BA

4. (24257-!-item-!-188;#058&001200)
Which of the following most logically completes the argument?
A new machine for harvesting corn will allow rows to be planted only fifteen inches apart, instead of the usual thirty inches. Corn planted this closely will produce lower yields per plant. Nevertheless, the new machine will allow corn growers to double their profits per acre because __________.
(A) with the closer spacing of the rows, the growing corn plants will quickly form a dense canopy of leaves, which will, by shading the ground, minimize the need for costly weed control and irrigation

In this case, it points out an advantage that closer planting has. Since the need for costly weed control and irrigation is minimized, the profits will increase. BA
(B) with the closer spacing of the rows, corn plants will be forced to grow taller because of increased competition for sunlight from neighboring corn plants

Actually, this weakens the argument. Growing taller will lower yields per plant.
(C) with the larger number of plants growing per acre, more fertilizer will be required

This is similar to B.
(D) with the spacing between rows cut by half, the number of plants grown per acre will almost double


Even though the number of plants doubles, the profits won’t make sure to increase per acre, considering the lower yields per plant.
(E) with the closer spacing of the rows, the acreage on which corn is planted will be utilized much more intensively than it was before, requiring more frequent fallow years in which corn fields are left unplanted

This does actually cast doubt on the conclusion.

作者: pennyz    时间: 2013-2-21 21:31
1:17s
e
premise: today's newspaper tell more about crime and violence than before
conclusion: today is not so safe and we should avoid go out
rephrase: maybe the coverage increase but actually case is not upward
(A) Newspapers and televised news programs have
more comprehensive coverage of violent
crime than newspapers did in the old days.------comprehesive about crime is out of scoupe
(B) National data show that violent crime is out of
control everywhere, not just in the author’s
city.-------support
(C) Police records show that people experience
more violent crimes in their own
neighborhoods than they do outside their
neighborhoods.--------compare with the area outside the neighbor,irrelevant
(D) Murder comprised a larger proportion of
violent crimes in the old days than it does
today.--------the specific category in the crime is out of scoupe
(E) News magazines play a more important role
today in informing the public about crime
than they did in the old days.-----correct
一定要把选项看完!还有不要胡乱意会意思
作者: pennyz    时间: 2013-2-21 22:22
2:05sexplain
premise:
1. in the urban,house replace mourning
2. in the rural ,the number is quiet stable
A(感觉a也不太对,但好像只有这个相关可以解释urban&rural的区别
2:01s
weaken
c
premise:v downtown bussiness small
       take some measures( lower tax rate)
conclusion:less likely to successs
rephrase:the shortcomings in downtown overwhelm its strengths
        suburbs has some advantages

1:49s
weaken
e
premise: now advantage: economy&turnover
        skybuse is fuel-efficient
conclusion:in the question stem
rephrase: skybuse may affect the turnover
1:19s
d
premise: machine planting cause half distance between the crops
        the shorten distance  cause less production
conclusion:the  harvast is still more
rephrase:because the number of crop is much more
作者: Elisha728    时间: 2013-9-4 07:04
8'11''
ADCED
作者: Elisha728    时间: 2013-9-4 07:06
balapupu 发表于 2011-12-9 08:52
逻辑链第四题做错了额。。。。。现在想想是自己粗心了~~(A) with the closer spacing of the rows, the gro ...

Thank you~
作者: Elisha728    时间: 2013-9-7 10:13
4 (24257-!-item-!-188;#058&001200)

Which of the following most logically completes the argument?


A new machine for harvesting corn will allow rows to be planted only fifteen inches apart, instead of the usual thirty inches.  Corn planted this closely will produce lower yields per plant.  Nevertheless, the new machine will allow corn growers to double their profits per acre because __________.


(A) with the closer spacing of the rows, the growing corn plants will quickly form a dense canopy of leaves, which will, by shading the ground, minimize the need for costly weed control and irrigation. Thus lower the cost and increase the profit.
(B) with the closer spacing of the rows, corn plants will be forced to grow taller because of increased competition for sunlight from neighboring corn plants. Irrelevant
(C) with the larger number of plants growing per acre, more fertilizer will be required. Irrelevant
(D) with the spacing between rows cut by half, the number of plants grown per acre will almost double.
This is the most attractive wrong answer. The passage already indicates that corns planted this closely will product lower yields per plant. So even if the number of plants per acre will almost double, there is no guarantee that the profits will double.
(E) with the closer spacing of the rows, the acreage on which corn is planted will be utilized much more intensively than it was before, requiring more frequent fallow years in which corn fields are left unplanted. Irrelevant
作者: lyrsilvia    时间: 2013-9-21 18:58
C DCEA
1. 21‘
P: more report=more murders and assaults
C:  to be safe from personal attack, one should not leave one’s home except for absolute necessities.
DOUBT THE CONCLUSION: at home≠safe
答案:C
(A) Newspapers and televised news programs have more comprehensive coverage of violent crime than newspapers did in the old days.-------------maybe correct but not doubt the conclusion CORRECT
(B) National data show that violent crime is out of control everywhere, not just in the author’s city.-------------where the crime is out of control is not doubt the conclusion
(C) Police records show that people experience more violent crimes in their own neighborhoods than they do outside their neighborhoods.-----------------CORRECT. means at home ≠safer
(D) Murder comprised a larger proportion of violent crimes in the old days than it does today.----------what murders comprised is irrelevant
(E) News magazines play a more important role today in informing the public about crime than they did in the old days.------------what the role of new magazines does not in this issue
===============================================================
C项还是在说in the neighborhood,还是outside the house,错误理解成了in the house
A项意思表明report more≠actually more murders or assaults.过分执著与doubt the conclusion,其实这里A项明显 doubt the premise,从而自然就doubt the conclusion. means people is safe now, do not need to stay in house
===============================================================

2. 34'
P: populations of the two species are more stable in rural areas far from human settlement; house gecko does not attack the mourning gecko, but in areas where insects congregate it prevents the mourning gecko from feeding on them
C: a newly house gecko is displacing the previously established mourning gecko in urban areas
EXPLAIN: urban areas have more insects congregate
答案:D
(A) In urban areas, geckos are valued because they eat so many insects.--------how the geckos are valued is irrelevant
(B) Geckos defend territories against other members of the same species.----------what geckos do for territories does not matter at all
(C) House geckos that arrive on islands are carried there in boats and planes.-----------how the geckos are carried is irrelevant
(D) In places where there are lighted buildings, insects tend to gather around the light.------------CORRECT. imply in urban areas, more lighted buildings cause more insects gather around. and explain the replacement of house gecko
(E) Mourning geckos are all females and reproduce asexually, but house geckos reproduce sexually.-------------how they reproduce is irrelevant.

3. 24'
BG: most Villieu-area businesses are now located only in the suburbs.The office buildings downtown lack the modern amenities most business operators demand today
P: have several modern office buildings constructed and to offer reduced local tax rates to any business that leases space in a new downtown building.
C:  lure more businesses downtown
Threaten the plan's SUCCESS: modern office building and reduced tax rates are not sufficient to attract business; suburbs have more attractions
答案:C
(A) Most of the businesses currently located in downtown Villieu have long-term leases on the space they occupy there.------------whether businesses have long-term leases, do not affect the plan or the tax and modern factors
(B) The existing office buildings in downtown Villieu have, on average, a much higher vacancy rate than do office buildings in Villieu's suburbs.-----------------vacancy rate is not said to be a factor that business considers
(C) The local tax rates in Villieu's suburbs are significantly lower than downtown Villieu's proposed rate for businesses that lease space in the new office buildings.-------------CORRECT. means significantly lower rate lure more businesses than downtown
(D) Most of the businesses that currently lease office space in downtown Villieu also lease office space in Villieu's suburbs.----------------where the businesses locate their office space does not clearly affect the plan.
(E) With the new office buildings, downtown Villieu would have a greater amount of modern office space than any other downtown business district in the region.----------support the success of the plan

4. 29'
BG: Northern Air has dozens of flights daily into and out of Belleville Airport, which is highly congested.
      Northern Air depends for its success on economy and quick turnaround
PLAN: replace its large planes with Skybusess, whose novel aerodynamic design is extremely fuel efficient.  The Skybus' fuel efficiency results in both lower fuel costs and reduced time spent refueling.
DISADVANTAGE OF THE PLAN: replace cause problems
答案:E
(A) The Skybus would enable Northern Air to schedule direct flights to destinations that currently require stops for refueling.--------------advantages
(B) Aviation fuel is projected to decline in price over the next several years.---------fuel price is irrelevant
(C) The fuel efficiency of the Skybus would enable Northern Air to eliminate refueling at some of its destinations, but several mechanics would lose their jobs.----------jobs or employment is not considered
(D) None of Northern Air's competitors that use Belleville Airport are considering buying Skybuses.--------------whether competitors are buying Skybuses does not affect. and because of the advantage of Skybuses, this statement is slightly support the success
(E) The aerodynamic design of the Skybus causes turbulence behind it when taking off that forces other planes on the runway to delay their takeoffs.--------------CORRECT. means delay will appear, this is the disadvantage of "quick turnaround".

5. 24'
P: Corn planted this closely will produce lower yields per plant
C:  the new machine will allow corn growers to double their profits per acre
EXPLAIN: profits can generate from other factors which can compensate for the loss of per plant yield
答案:A
(A) with the closer spacing of the rows, the growing corn plants will quickly form a dense canopy of leaves, which will, by shading the ground, minimize the need for costly weed control and irrigation--------------CORRECT. means advantages for reduce cost
(B) with the closer spacing of the rows, corn plants will be forced to grow taller because of increased competition for sunlight from neighboring corn plants-----------------taller does not mean more yield. this statement is meaningless.
(C) with the larger number of plants growing per acre, more fertilizer will be required-------------clearly states the disadvantage and the increased cost.
(D) with the spacing between rows cut by half, the number of plants grown per acre will almost double-----------number of plants double does not mean the yield will doubt. thus insufficient to generate
(E) with the closer spacing of the rows, the acreage on which corn is planted will be utilized much more intensively than it was before, requiring more frequent fallow years in which corn fields are left unplanted. ------------clear disadvantages. not support the double profits.
作者: 览物之情    时间: 2013-10-19 22:59
10月19日
1 19s
Premise: Unlike newspapers in the old days, today’s newspapers and televised news programs are full of stories about murders and assaults in our city.  Conclusion:One can only conclude from this change that violent crime is now out of control, and, to be safe from personal attack, one should not leave one’s home except for absolute necessities.

Which one of the following, if true, would cast the
most serious doubt on the conclusion?
P AB类比what's the difference between today’s N and old day’s N. maybe N today are more willing to write about crime.
(A) Newspapers and televised news programs have more comprehensive coverage of violent crime than newspapers did in the old days.___correct
(B) National data show that violent crime is out ofcontrol everywhere, not just in the city.
(C) Police records show that people experience more violent crimes in their own
neighborhoods than they do outside theirneighborhoods.
(D) Murder comprised a larger proportion ofviolent crimes in the old days than it does
today.
(E) News magazines play a more important role today in informing the public about crime
than they did in the old days.


2 38s
Background:On Pacific islands, a newly arrived gecko species, the house gecko, is displacing the previously established mourning gecko in urban areas,
Premise: populations of the two species are more stable in rural areas far from human settlement.  
Premise2:The house gecko does not attack the mourning gecko, but in areas where insects congregate it prevents the mourning gecko from feeding on them.

Which of the following contributes most to an explanation of the difference between gecko populations in urban and rural areas?
现象解释: what’s the relationship between human and insects congregate.
(A) In urban areas, geckos are valued because they eat so many insects.
(B) Geckos defend territories against other members of the same species.
(C) House geckos that arrive on islands are carried there in boats and planes.
(D) In places where there are lighted buildings, insects tend to gather around the light.---correct
(E) Mourning geckos are all females and reproduce asexually, but house geckos reproduce sexually.

3 35s
Backgroundowntown Villieu was once a flourishing business district, but most Villieu-area businesses are now located only in the suburbs.(郊区)
Premise: The office buildings downtown lack the modern amenities most business operators demand today.  
Plan: To lure more businesses downtown, Villieu officials plan to have several modern office buildings constructed and to offer reduced local tax rates to any business that leases space in a new downtown building.

Which of the following, if true, most threatens the plan's likelihood of success?
可行性。What if business today need large area.

(A) Most of the businesses currently located in downtown Villieu have long-term leases on the space they occupy there.----irrelevant.因为是新建的所以之前的没影响
(B) The existing office buildings in downtown Villieu have, on average, a much higher vacancy rate than do office buildings in Villieu's suburbs.----background
(C) The local tax rates in Villieu's suburbs are significantly lower than downtown Villieu's proposed rate for businesses that lease space in the new office buildings.-----correct
(D) Most of the businesses that currently lease office space in downtown Villieu also lease office space in Villieu's suburbs.----irrelevant
(E) With the new office buildings, downtown Villieu would have a greater amount of modern office space than any other downtown business district in the region.---irrelevant.


4 29s
Background:Northern Air has dozens of flights daily into and out of Belleville Airport, which is highly congested.  
Premise: Northern Air depends for its success on economy and quick turnaround and
plan: consequently is planning to replace its large planes with Skybuses, whose novel aerodynamic design is extremely fuel efficient.  The Skybus' fuel efficiency results in both lower fuel costs and reduced time spent refueling.

Which of the following, if true, could present the most serious disadvantage for Northern Air in replacing their large planes with Skybuses?副作用

(A) The Skybus would enable Northern Air to schedule direct flights to destinations that currently require stops for refueling.
(B) Aviation fuel is projected to decline in price over the next several years.
(C) The fuel efficiency of the Skybus would enable Northern Air to eliminate refueling at some of its destinations, but several mechanics would lose their jobs.
(D) None of Northern Air's competitors that use Belleville Airport are considering buying Skybuses.
(E) The aerodynamic design of the Skybus causes turbulence behind it when taking off that forces other planes on the runway to delay their takeoffs.-----correct.

5 22s must be true类型
Which of the following most logically completes the argument?
A new machine for harvesting corn will allow rows to be planted only fifteen inches apart, instead of the usual thirty inches.  Corn planted this closely will produce lower yields per plant.  Nevertheless, the new machine will allow corn growers to double their profits per acre because __________. Must be true.

种的更密。
(A) with the closer spacing of the rows, the growing corn plants will quickly form a dense canopy of leaves, which will, by shading the ground, minimize the need for costly weed control and irrigation----correct
(B) with the closer spacing of the rows, corn plants will be forced to grow taller because of increased competition for sunlight from neighboring corn plants
(C) with the larger number of plants growing per acre, more fertilizer will be required。
(D) with the spacing between rows cut by half, the number of plants grown per acre will almost double---干扰项。题干已经说了产量是减少
(E) with the closer spacing of the rows, the acreage on which corn is planted will be utilized much more intensively than it was before, requiring more frequent fallow years in which corn fields are left unplanted

作者: w.melhere    时间: 2013-11-2 13:14
错两道题....

1.23‘  54‘  错了。开始没有理解对comprehensive coverage的意思。
A突出了 more comprehensive ,解释了是newspaper加大了范围而不是社会out of control——即使同样的犯罪率,也可能因为跟全面的报道而增多数量。而E中关注的是角色更重要,跑偏。

2.  1’33 题目的最后一句话没看懂。后来反复看了好几遍才理顺...
“The house gecko does not attack the mourning gecko, but in areas where insects congregate it prevents the mourning gecko from feeding on them.”这句话的作用在于解释前一句两地数量差异大。是说HG不攻击MG(这不是数量相差的原因),但MG会因为insects密集,无法捕食而数量减少。所以在城市和郊区的G数量相差很大。D选项解释了为什么城市里insects密集。

3.45' 1.28'
4.39'  1.26'
5. 1.15'
发现自己做题时间其实还有剩余,可以再放慢一点,一遍没有理顺的再读一遍。
作者: yuehuasunday    时间: 2013-11-21 16:33
1-19

1.

Background: Unlike newspapers in the old days, today’s
newspapers and televised news programs are full of
stories about murders and assaults in our city.

Premise: One
can only conclude from this change that violent
crime is now out of control, and, to be safe from
personal attack,

Conclusion: one should not leave one’s home
except for absolute necessities.

(A) Correct

2.

Background: On Pacific islands, a newly arrived gecko species, the house gecko, is displacing the previously established mourning gecko in urban areas

Premise: but populations of the two species are more stable in rural areas far from human settlement.

Conclusion: The house gecko does not attack the mourning gecko, but in areas where insects congregate it prevents the mourning gecko from feeding on them.

(D) correct

3.

Background: Downtown Villieu was once a flourishing business district, but most Villieu-area businesses are now located only in the suburbs.

Premise: The office buildings downtown lack the modern amenities most business operators demand today.

Conclusion: To lure more businesses downtown, Villieu officials plan to have several modern office buildings constructed and to offer reduced local tax rates to any business that leases space in a new downtown building.

© Correct

4.

Background: Northern Air has dozens of flights daily into and out of Belleville Airport, which is highly congested.

Premise: Northern Air depends for its success on economy and quick turnaround and consequently is planning to replace its large planes with Skybuses, whose novel aerodynamic design is extremely fuel efficient.

Conclusion: The Skybus' fuel efficiency results in both lower fuel costs and reduced time spent refueling.

(E)

5.

Background: A new machine for harvesting corn will allow rows to be planted only fifteen inches apart

Premise: instead of the usual thirty inches

Conclusion:  Corn planted this closely will produce lower yields per plant.  Nevertheless, the new machine will allow corn growers to double their profits per acre because __________.

(A) Correct

作者: irenetopia    时间: 2013-12-2 14:27
【精练】

1:01
6. Unlike newspapers in the old days, today’s
newspapers and televised news programs are full of
stories about murders and assaults in our city. One
can only conclude from this change that violent
crime is now out of control, and, to be safe from
personal attack, one should not leave one’s home
except for absolute necessities.
Which one of the following, if true, would cast the
most serious doubt on the conclusion?
逻辑链:报纸和电视在当前比过去播放(刊登)更多的关于谋杀之类的文章
因为现在的暴力犯罪更多了,为了要让人有防范意识
问削弱
思路:报道多——因为时间多
找出报道和时间因果关系问题或者他因导致,也就是并不是暴力犯罪多了的问题

(A) Newspapers and televised news programs have
more comprehensive coverage of violent
crime than newspapers did in the old days.——比过去更全面的报道,跟
(B) National data show that violent crime is out of
control everywhere, not just in the author’s
city.——support
(C) Police records show that people experience
more violent crimes in their own
neighborhoods than they do outside their
neighborhoods. ——support
(D) Murder comprised a larger proportion of
violent crimes in the old days than it does
today.——irrelevant
(E) News magazines play a more important role
today in informing the public about crime
than they did in the old days.——bingo!因为扮演角色更重要,所以篇幅就多了,weaken


【逻辑链】
1.    50S
On Pacific islands, a newly arrived gecko species, the house gecko, is displacing the previously established mourning gecko in urban areas, but populations of the two species are more stable in rural areas far from human settlement.  The house gecko does not attack the mourning gecko, but in areas where insects congregate it prevents the mourning gecko from feeding on them.


Which of the following contributes most to an explanation of the difference between gecko populations in urban and rural areas?
逻辑链:新来的G种类和之前的G种类和平相处,
但是在昆虫多的地方,新来的G却阻止之前的MG捕食昆虫
问解释:
应该是跟昆虫有关对二者的影响
(A) In urban areas, geckos are valued because they eat so many insects.—说了G的性质,无关
(B) Geckos defend territories against other members of the same species.——没有,前提都说了他们相处的不错
(C) House geckos that arrive on islands are carried there in boats and planes.——G到来的方式,无关
(D) In places where there are lighted buildings, insects tend to gather around the light.——bingo!可能是因为光的原因造成的
(E) Mourning geckos are all females and reproduce asexually, but house geckos reproduce sexually.
——传播因素跟攻击无关

2.    1:19


Downtown Villieu was once a flourishing business district, but most Villieu-area businesses are now located only in the suburbs.  The office buildings downtown lack the modern amenities most business operators demand today.  To lure more businesses downtown, Villieu officials plan to have several modern office buildings constructed and to offer reduced local tax rates to any business that leases space in a new downtown building.


Which of the following, if true, most threatens the plan's likelihood of success?
逻辑链:V的商区现在都在乡下,现在office想让大家都来downtown发展事业
于是他们:盖楼,减税率
问削弱
思路:1目标达不到 2 副作用,即使减税盖楼也可能失败,应该是还有什么是suburb有的优势
(A) Most of the businesses currently located in downtown Villieu have long-term leases on the space they occupy there.——irrelevant吧
(B) The existing office buildings in downtown Villieu have, on average, a much higher vacancy rate than do office buildings in Villieu's suburbs.——空缺率是好事儿啊
(C) The local tax rates in Villieu's suburbs are significantly lower than downtown Villieu's proposed rate for businesses that lease space in the new office buildings.——bingo!suburb的税比downtown减了以后的税率还低!
(D) Most of the businesses that currently lease office space in downtown Villieu also lease office space in Villieu's suburbs.——irrelevant
(E) With the new office buildings, downtown Villieu would have a greater amount of modern office space than any other downtown business district in the region.——support


3.   49S

Northern Air has dozens of flights daily into and out of Belleville Airport, which is highly congested.  Northern Air depends for its success on economy and quick turnaround and consequently is planning to replace its large planes with Skybuses, whose novel aerodynamic design is extremely fuel efficient.  The Skybus' fuel efficiency results in both lower fuel costs and reduced time spent refueling.

Which of the following, if true, could present the most serious disadvantage for Northern Air in replacing their large planes with Skybuses?
逻辑链:因为NA机场拥堵,为了提高经济效益喝速度,他们决定用S代替大型客机
S的燃料成本低而且加油时间短
问削弱
思路:首先抓住关键,为了么要换,因为congested!
所以后面说了S的一堆特点,其实不是很重要,重要的是是不是能够达到reduce congested
(A) The Skybus would enable Northern Air to schedule direct flights to destinations that currently require stops for refueling.——support
(B) Aviation fuel is projected to decline in price over the next several years.——irrelevant
(C) The fuel efficiency of the Skybus would enable Northern Air to eliminate refueling at some of its destinations, but several mechanics would lose their jobs.——irrelevant
(D) None of Northern Air's competitors that use Belleville Airport are considering buying Skybuses.——irrelevant
(E) The aerodynamic design of the Skybus causes turbulence behind it when taking off that forces other planes on the runway to delay their takeoffs.——bingo!delay!!!目标达不到啊


4.   47S
Which of the following most logically completes the argument?
A new machine for harvesting corn will allow rows to be planted only fifteen inches apart, instead of the usual thirty inches.  Corn planted this closely will produce lower yields per plant.  Nevertheless, the new machine will allow corn growers to double their profits per acre because __________.
逻辑链:新机器种的玉米比原来的少了15inchs,yield就少了
但是新机器可以带来更高的利润
问为什么:思路路:销量——成本=利润
要想产量少,利润却多,只能在成本上做文章了,大概关于降低成本的选项或者可能corn最近忽然涨价了

(A) with the closer spacing of the rows, the growing corn plants will quickly form a dense canopy of leaves, which will, by shading the ground, minimize the need for costly weed control and irrigation——bingo!cost少了!
(B) with the closer spacing of the rows, corn plants will be forced to grow taller because of increased competition for sunlight from neighboring corn plants——长得更高没有用啊
(C) with the larger number of plants growing per acre, more fertilizer will be required——这是weaken吧
(D) with the spacing between rows cut by half, the number of plants grown per acre will almost double——这样应该是持平?
(E) with the closer spacing of the rows, the acreage on which corn is planted will be utilized much more intensively than it was before, requiring more frequent fallow years in which corn fields are left unplanted——irrelevant




作者: cyndichiang    时间: 2014-4-18 13:40
精练题 33''
1.premise: today’s newspapers and televised news programs are full of stories about murders and assaults in our city
conclusion: violent crime is now out of control, and, to be safe from personal attack, one should not leave one’s home except for absolute necessities
Q:weaken
prephrase: newspapers and televised news purposely report something dangerous and repeat some cases over and over again,thus actually violent cases appear not as frequently as  the news reported.
(A)Newspapers and televised news programs have
more comprehensive coverage of violent
crime than newspapers did in the old days.---correct,since it gives another factors that leads to the high violent news,namely the more comprehensive converge. so we can infer that the increased violent news are not attributed to the violent crimes themselves.
(B) National data show that violent crime is out of
control everywhere, not just in the author’s
city.----opposite ,this statement strengthen the conclusion that we should not go out of home
(C) Police records show that people experience
more violent crimes in their own
neighborhoods than they do outside their
neighborhoods.------irrelevant,since wherever the more violent crimes happen, it is dangerous to get out of home,thus this statement is slightly strengthen.
(D) Murder comprised a larger proportion of
violent crimes in the old days than it does
today.------irrelevant, what kind of violent crimes is irrelevant to the point.注意这里是相对数与绝对数的比较,murder is a proportion of violent crimes,so if we don't know what proportion of murder occupies the total violent crimes,the statement is unclear.
(E) News magazines play a more important role
today in informing the public about crime
than they did in the old days.---我原本以为new magazines play a more important role表示数据可能不真实,可能反复播出故意引起人们注意。文章中并没有提这方面的信息,所以不知道more informing代表着什么,因此是irrelevant 注意!不要自己过度推理要基于原文的信息
作者: cyndichiang    时间: 2014-4-18 14:08
逻辑链
1.35‘’
premise:  the house gecko, is displacing the previously established mourning gecko in urban areas
premise:The house gecko does not attack the mourning gecko, but in areas where insects congregate it prevents the mourning gecko from feeding on them.
conclusion: populations of the two species are more stable in rural areas far from human settlement.
Q: explain
prephrase: in rural area, insects are less likely to congregate thus the number of mourning gecko is decreasing

2. 41''
BG: most Vi businesses are now located only in the suburbs and the office buildings downtown lack the modern amenities most business operators demand today.
premise: V officials plan to have several modern office buildings constructed and to offer reduced local tax rates to any business that leases space in a new downtown building
conclusion: this plan can successfully attract business
Q:support
prephrase: support a plan: 1. deny a side-effect of the plan
                                the price or lease bill of the new building will not much higher than the current ones
                               there is no difference about the taxes between downtown and suburb

3.BG: Skybuse is  novel aerodynamic design and extremely fuel efficient and can quickly turnaround
premise: NA plans to replace its large plane with skybuse
conclusion: NA can reduce time of refueling thus the congestion and cost
Q:weaken
prephrase: cost of replacement is extremely high
              when refueling, the skybuse may cost more time without the help of large planes


4. 27''
premise: planting corn rows only fifteen inches apart instead of the usual thirty inches will lead to the decline of production
premise:???
conclusion: the method will allow corn growers to double their profits per acre
Q: support
prephrase: the cost of new method will reduce a lot


订正:第一题逻辑链错了,看了选项之后发现时要解释为什么the house gecko is displacing the previously established mourning gecko in urban areas



作者: simonleo    时间: 2014-5-12 18:44
1.
Time:33s
P:unlike newspapers in the old day, today's newspaper and televised news programs are full of stories about murders and assauts.
C: violent crime is now out of control, and, to be safe from personal attack, one should not leave one’s home except for absolute necessities.
Pre:削弱 报道量变多只能说明现在报道的比以前多,不代表犯罪量就真的变多了
(A) Newspapers and televised news programs have
more comprehensive coverage of violent
crime than newspapers did in the old days. 正解
(B) National data show that violent crime is out of
control everywhere, not just in the author’s
city.
(C) Police records show that people experience
more violent crimes in their own
neighborhoods than they do outside their
neighborhoods.
(D) Murder comprised a larger proportion of
violent crimes in the old days than it does
today.
(E) News magazines play a more important role
today in informing the public about crime
than they did in the old days.注意该选项 说magazine的地位问题,不能直接反应是不是报道数量比以前多了

2. 1:00 D 不是特别清晰 只是觉得D指出了城市与乡村的差别 A选项没有体现两者的差别。
3. 50s C
4. 48s E
5. 41s D A为什么不行? 。。。正确答案是A。。。 那D为什么不行。。。 这么去理解,profits要double 首先你per plant的产量下降,即使你per acre的plants数量double了,你总产量还是没有double的,那么不能解释profits double这一现象。 A就可以解释,因为cost变少了 一定要注意这个数量关系。
作者: MIA926    时间: 2015-9-28 19:52

【精练】
6. Unlike newspapers in the old days, today’s
newspapers and televised news programs are full of
stories about murders and assaults in our city. One
can only conclude from this change that violent
crime is now out of control, and, to be safe from
personal attack, one should not leave one’s home
except for absolute necessities.
逻辑链:如今的新闻到处都是关于谋杀等消息。所以人们可以得出结论,如今的暴力犯罪不受控制,人们应该减少不必要的出行
推测:新闻报道的多和实际多还是有很大差距的
Which one of the following, if true, would cast the
most serious doubt on the conclusion?
(A) Newspapers and televised news programs have
more comprehensive coverage of violent
crime than newspapers did in the old days. 无关
(B) National data show that violent crime is out of
control everywhere, not just in the author’s
city. 加强
(C) Police records show that people experience
more violent crimes in their own
neighborhoods than they do outside their
neighborhoods. 无关
(D) Murder comprised a larger proportion of
violent crimes in the old days than it does
today. 无关
(E) News magazines play a more important role
today in informing the public about crime
than they did in the old days. 正确

【逻辑链】
1.    (24021-!-item-!-188;#058&000691)
On Pacific islands, a newly arrived gecko species, the house gecko, is displacing the previously established mourning gecko in urban areas, but populations of the two species are more stable in rural areas far from human settlement.  The house gecko does not attack the mourning gecko, but in areas where insects congregate it prevents the mourning gecko from feeding on them.
逻辑链:在太平洋的岛上,HG替代了原有的MG在城市地带;但是在乡村地区则没有太大变化。HG不攻击MG,但是会组织MG吃一堆虫子
推测:城市只有一堆虫子,要吃的时候老是被阻止然后就死完了。。。

Which of the following contributes most to an explanation of the difference between gecko populations in urban and rural areas?


(A) In urban areas, geckos are valued because they eat so many insects. 新消息
(B) Geckos defend territories against other members of the same species.  新消息
(C) House geckos that arrive on islands are carried there in boats and planes. 新消息
(D) In places where there are lighted buildings, insects tend to gather around the light. 正确
(E) Mourning geckos are all females and reproduce asexually, but house geckos reproduce sexually. 无关

2.    (24069-!-item-!-188;#058&000703)
Downtown Villieu was once a flourishing business district, but most Villieu-area businesses are now located only in the suburbs.  The office buildings downtown lack the modern amenities most business operators demand today.  To lure more businesses downtown, Villieu officials plan to have several modern office buildings constructed and to offer reduced local tax rates to any business that leases space in a new downtown building.
逻辑链:DV原来是商业中心,但是现在商业区挪到了郊区。市中心的办公区缺乏如今很多商业活动需要的设施。为了吸引公司到市中心,官员计划建设更多的现代化办公楼,并为那些在市中心租房的公司提供更低的税率
推测:方案题达不到目的一个是可能没人响应,一个是有不良后果
Which of the following, if true, most threatens the plan's likelihood of success?
(A) Most of the businesses currently located in downtown Villieu have long-term leases on the space they occupy there. 无关
(B) The existing office buildings in downtown Villieu have, on average, a much higher vacancy rate than do office buildings in Villieu's suburbs. 无关
(C) The local tax rates in Villieu's suburbs are significantly lower than downtown Villieu's proposed rate for businesses that lease space in the new office buildings. 正确
(D) Most of the businesses that currently lease office space in downtown Villieu also lease office space in Villieu's suburbs. 无关
(E) With the new office buildings, downtown Villieu would have a greater amount of modern office space than any other downtown business district in the region. 可能加强


3.    (24117-!-item-!-188;#058&000708)
Northern Air has dozens of flights daily into and out of Belleville Airport, which is highly congested.  Northern Air depends for its success on economy and quick turnaround and consequently is planning to replace its large planes with Skybuses, whose novel aerodynamic design is extremely fuel efficient.  The Skybus' fuel efficiency results in both lower fuel costs and reduced time spent refueling.
逻辑:NA每天有很多航班,然后highly congested。NA的成功基于经济和quick turnaround,他计划用S替换大飞机。S燃油效率高,使得油费下降而且减少了时间
推测:反正就是讲S不好

Which of the following, if true, could present the most serious disadvantage for Northern Air in replacing their large planes with Skybuses?


(A) The Skybus would enable Northern Air to schedule direct flights to destinations that currently require stops for refueling. 加强
(B) Aviation fuel is projected to decline in price over the next several years. 无关
(C) The fuel efficiency of the Skybus would enable Northern Air to eliminate refueling at some of its destinations, but several mechanics would lose their jobs. 前加强 后无关
(D) None of Northern Air's competitors that use Belleville Airport are considering buying Skybuses. 无关
(E) The aerodynamic design of the Skybus causes turbulence behind it when taking off that forces other planes on the runway to delay their takeoffs. 正确


4.    (24257-!-item-!-188;#058&001200)
Which of the following most logically completes the argument?
A new machine for harvesting corn will allow rows to be planted only fifteen inches apart, instead of the usual thirty inches.  Corn planted this closely will produce lower yields per plant.  Nevertheless, the new machine will allow corn growers to double their profits per acre because __________.
逻辑:新机器可以使农作物种植距离减少一半,但是种的这么近农作物会减产。尽管如此,新机器还是会使农民受益加倍
推测:种的多或者是有其他好处,可以是C减少或是r增加
P=r-c

(A) with the closer spacing of the rows, the growing corn plants will quickly form a dense canopy of leaves, which will, by shading the ground, minimize the need for costly weed control and irrigation 虽然支持了结论 但是没有讲到倍数的问题
(B) with the closer spacing of the rows, corn plants will be forced to grow taller because of increased competition for sunlight from neighboring corn plants 长得高和profit关系还是蛮远的
(C) with the larger number of plants growing per acre, more fertilizer will be required 削弱
(D) with the spacing between rows cut by half, the number of plants grown per acre will almost double 正确 主要是结论里面还有倍数
(E) with the closer spacing of the rows, the acreage on which corn is planted will be utilized much more intensively than it was before, requiring more frequent fallow years in which corn fields are left unplanted 无关

作者: spencerchan    时间: 2016-1-29 11:27
47’’
P: news reports more crimes than before
C: Security is worse than before
C: People should not leave homes
Weaken: In the past, newspaper did not report crimes even if it happened/people also cannot protect themselves even they stay at home
A: Comprehensive coverage??
B: Irrelevant
C: Irrelevant
D: Irrelevant
E: CORRECT


1’25’’
P: Urban: H has already displaced M
P: Insects congregate=>M cannot feed on H
P: H does not attack M
C: in rural areas H&M are stable
A: Irrelevant
B: Irrelevant
C: Irrelevant
D: CORRECT?
E: Irrelevant

43’’
P: Downtown Villein lacks the modern amenities
P: Villieu decides build modern office buildings and reduce tax rate
C: V would improve local businesses
Weaken: Modern amenities are not equal to modern office building/There are other reasons that businesses do not want to be located in V
A: Irrelevant
B: Irrelevant
C: CORRECT
D: Irrelevant
E: Strengthen

31’’
P: Skybus=> lower fuel costs+time-saving
P: Currently, N air has quick turnaround
C: replacing old plants with Skybus could bring benefits
Weaken: Skybus has serious weakness
A: Strengthen
B: Irrelevant
C: Strengthen
D: Irrelevant
E: CORRECT/WEAKEN

59’’
P: new machine will allow rows to be planted closer
P: yields per plant will be lower
C: Profit/acre will be boost
Since: Farmer can plant more corns than before
A: Flaw: profit????
B: Weaken
C: Weaken
D: CORRECT???
E: Weaken

错的两题都在两个选项里纠结,结果都选了错的那一个。。。
作者: fesche    时间: 2016-5-20 05:22
news broadcast murder
crimes out of control
one can be safe at home
doubt
因:电视和新闻报道太多谋杀的新闻
果:犯罪升高,在家安全
E-A
(A) Newspapers and televised news programs have
more comprehensive coverage of violent
crime than newspapers did in the old days.
(B) National data show that violent crime is out of
control everywhere, not just in the author’s
city.
(C) Police records show that people experience
more violent crimes in their own
neighborhoods than they do outside their
neighborhoods.
(D) Murder comprised a larger proportion of
violent crimes in the old days than it does
today.
(E) News magazines play a more important role
today in informing the public about crime
than they did in the old days.

H is displacing M in urban area
in rural HM no changed
in insect congregate H prevents M from feeding on them
为什么rural和urban不一样
在没有insect的地方H会抢M的
B,C 都有可能 选B
(A) In urban areas, geckos are valued because they eat so many insects.
(B) Geckos defend territories against other members of the same species.
(C) House geckos that arrive on islands are carried there in boats and planes.
(D) In places where there are lighted buildings, insects tend to gather around the light.
(E) Mourning geckos are all females and reproduce asexually, but house geckos reproduce sexually.

downtown once was flourished
now business in suburb
lack of amenities
build new buildings, lower tax to lure more business
目的: 吸引business
weaken
因果,因不导致果,他因
人口在郊外,生意都是跟着住的人走
C
(A) Most of the businesses currently located in downtown Villieu have long-term leases on the space they occupy there.
(B) The existing office buildings in downtown Villieu have, on average, a much higher vacancy rate than do office buildings in Villieu's suburbs.
(C) The local tax rates in Villieu's suburbs are significantly lower than downtown Villieu's proposed rate for businesses that lease space in the new office buildings.
(D) Most of the businesses that currently lease office space in downtown Villieu also lease office space in Villieu's suburbs.
(E) With the new office buildings, downtown Villieu would have a greater amount of modern office space than any other downtown business district in the region.

E

A
(A) with the closer spacing of the rows, the growing corn plants will quickly form a dense canopy of leaves, which will, by shading the ground, minimize the need for costly weed control and irrigation
(B) with the closer spacing of the rows, corn plants will be forced to grow taller because of increased competition for sunlight from neighboring corn plants
(C) with the larger number of plants growing per acre, more fertilizer will be required
(D) with the spacing between rows cut by half, the number of plants grown per acre will almost double
(E) with the closer spacing of the rows, the acreage on which corn is planted will be utilized much more intensively than it was before, requiring more frequent fallow years in which corn fields are left unplanted
作者: May97    时间: 2018-3-6 10:19
T:21s
P:today’s newspapers and televised news programs are full of stories about murders and assaults in our city.
C:One can only conclude from this change that violent crime is now out of control
Weaken:newspapers are more eager to report crime
(A) Newspapers and televised news programs have more comprehensive coverage of violent crime than newspapers did in the old days.--->R
(B) National data show that violent crime is out of control everywhere, not just in the author’s city.--->SUPPORT
(C) Police records show that people experience more violent crimes in their own neighborhoods than they do outside their neighborhoods.--->SUPPORT
(D) Murder comprised a larger proportion of violent crimes in the old days than it does today.--->IRREVERENT
(E) News magazines play a more important role today in informing the public about crime than they did in the old days.--->IRREVERENT

T: 37S
P: populations of the two species are more stable in rural areas far from human settlement.  
P: The house gecko does not attack the mourning gecko, but in areas where insects congregate it prevents the mourning gecko from feeding on them.
EP: IN HUMAN SETTLEMENT INSECT CONGREGATE
(A) In urban areas, geckos are valued because they eat so many insects.--->IRREVERENT
(B) Geckos defend territories against other members of the same species.--->NEW
(C) House geckos that arrive on islands are carried there in boats and planes.--->NEW
(D) In places where there are lighted buildings, insects tend to gather around the light.--->R
(E) Mourning geckos are all females and reproduce asexually, but house geckos reproduce sexually.--->NEW

T: 43S
BG: Downtown Villieu was once a flourishing business district, but most Villieu-area businesses are now located only in the suburbs.  
P: The office buildings downtown lack the modern amenities most business operators demand today.  
C: To lure more businesses downtown, Villieu officials plan to have several modern office buildings constructed and to offer reduced local tax rates to any business that leases space in a new downtown building.
Weaken:

作者: May97    时间: 2018-3-6 10:35
(A) Most of the businesses currently located in downtown Villieu have long-term leases on the space they occupy there.--->IRREVERENT
(B) The existing office buildings in downtown Villieu have, on average, a much higher vacancy rate than do office buildings in Villieu's suburbs.--->IRREVERENT
(C) The local tax rates in Villieu's suburbs are significantly lower than downtown Villieu's proposed rate for businesses that lease space in the new office buildings.--->R
(D) Most of the businesses that currently lease office space in downtown Villieu also lease office space in Villieu's suburbs.--->IRREVERENT
(E) With the new office buildings, downtown Villieu would have a greater amount of modern office space than any other downtown business district in the region.--->IRREVERENT

T: 43S
BG: Northern Air has dozens of flights daily into and out of Belleville Airport, which is highly congested.  
P: Northern Air depends for its success on economy and quick turnaround and consequently
P: The Skybus' fuel efficiency results in both lower fuel costs and reduced time spent refueling.
C: is planning to replace its large planes with Skybuses
WEAKEN: LOSE COUSTOMERS
(A) The Skybus would enable Northern Air to schedule direct flights to destinations that currently require stops for refueling.--->SUPPORT
(B) Aviation fuel is projected to decline in price over the next several years.--->IRREVERENT
(C) The fuel efficiency of the Skybus would enable Northern Air to eliminate refueling at some of its destinations, but several mechanics would lose their jobs.--->SUPPORT
(D) None of Northern Air's competitors that use Belleville Airport are considering buying Skybuses.--->IRREVERENT
(E) The aerodynamic design of the Skybus causes turbulence behind it when taking off that forces other planes on the runway to delay their takeoffs.--->R

T: 28S
P: Corn planted this closely will produce lower yields per plant.  
C: Nevertheless, the new machine will allow corn growers to double their profits per acre
SUPPORT: REDUCE COSTS FOR LABOR
(A) with the closer spacing of the rows, the growing corn plants will quickly form a dense canopy of leaves, which will, by shading the ground, minimize the need for costly weed control and irrigation--->R
(B) with the closer spacing of the rows, corn plants will be forced to grow taller because of increased competition for sunlight from neighboring corn plants--->IRREVERENT
(C) with the larger number of plants growing per acre, more fertilizer will be required--->IRREVERENT
(D) with the spacing between rows cut by half, the number of plants grown per acre will almost double--->WRONG
(E) with the closer spacing of the rows, the acreage on which corn is planted will be utilized much more intensively than it was before, requiring more frequent fallow years in which corn fields are left unplanted--->WEAKEN



作者: echo-LUO    时间: 2018-9-28 20:36
精练:
谋杀等新闻增加——>社会越来越险恶——>必须要在家带着才安全呢
A: 两条逻辑关系都不成立
(A) Newspapers and televised news programs have more comprehensive coverage of violent crime than newspapers did in the old days.
(B) National data show that violent crime is out of control everywhere, not just in the author’s city.
(C) Police records show that people experience more violent crimes in their own neighborhoods than they do outside their neighborhoods.
(D) Murder comprised a larger proportion o violent crimes in the old days than it does today.
(E) News magazines play a more important role today in informing the public about crime than they did in the old days.

逻辑链
1.
小岛上,h和m两种动物,h后来的,但两者数量都挺稳定——>但如果是在城市就不稳定了——>昆虫聚集的地方,h会不让m吃虫(m真可怜)
A:城市的昆虫很聚集
(A) In urban areas, geckos are valued because they eat so many insects.
(B) Geckos defend territories against other members of the same species.
(C) House geckos that arrive on islands are carried there in boats and planes.
(D) In places where there are lighted buildings, insects tend to gather around the light.
(E) Mourning geckos are all females and reproduce asexually, but house geckos reproduce sexually.


2.
D地本来商业中心在城里,现在都搬到了郊区——>城里缺少现代设施——>为了把商业拽回来,政府就想建现代建筑啊、降低税率呀
A:可能搬是因为其他原因吧,比如地租很贵,尽管政府给了一些福利,还是比在郊区呆着成本高
(A) Most of the businesses currently located in downtown Villieu have long-term leases on the space they occupy there. 感觉有可能,毕竟人家地没到期
瞎了...是downtown.....
(B) The existing office buildings in downtown Villieu have, on average, a much higher vacancy rate than do office buildings in Villieu's suburbs.
(C) The local tax rates in Villieu's suburbs are significantly lower than downtown Villieu's proposed rate for businesses that lease space in the new office buildings.
(D) Most of the businesses that currently lease office space in downtown Villieu also lease office space in Villieu's suburbs.
(E) With the new office buildings, downtown Villieu would have a greater amount of modern office space than any other downtown business district in the region.


3.
刚做过,不重复了

4.
一个新机器让种庄稼的间距缩短到15,原本要30——>缩短会使单位产出减少——>但是新机器可以让利润翻翻
(A) with the closer spacing of the rows, the growing corn plants will quickly form a dense canopy of leaves, which will, by shading the ground, minimize the need for costly weed control and irrigation
(B) with the closer spacing of the rows, corn plants will be forced to grow taller because of increased competition for sunlight from neighboring corn plants
(C) with the larger number of plants growing per acre, more fertilizer will be required
(D) with the spacing between rows cut by half, the number of plants grown per acre will almost double
(E) with the closer spacing of the rows, the acreage on which corn is planted will be utilized much more intensively than it was before, requiring more frequent fallow years in which corn fields are left unplanted









作者: 云栈    时间: 2019-11-9 07:55
1.        15s
P:与过去的报纸不同,今天的报纸和电视新闻节目中充斥着关于我们城市中谋杀和袭击的故事。
C: 暴力犯罪现在已经失控了,为了避免遭受人身伤害,除了绝对必需品,人们不应该离开家。
推测:现在的报到率比以前高了
选项分析:A
A:提出了另外一个原因
B:opposite 支持原题
C: 题上说的是murder 和assault,不是全部的violent crimes
D: 比例大不一定说明数量就大
E: 无关

2.        35s
P: 在太平洋岛屿上,一种新到的壁虎物种,即壁虎,正在取代以前在城市地区建立的哀悼壁虎,但在远离人类住区的农村地区,这两种物种的种群更加稳定。
P: 壁虎不会袭击丧葬的壁虎,但是在昆虫聚集的区域,它可以防止丧葬的壁虎以它们为食。
推测:在意外生存空间大,不需要竞争,可能还要互相支持成长;在城市生存空间小,需要竞争
选项分析:D
【没看懂】

3.        37s
P: 维尔留市中心曾经是一个繁华的商业区,但是现在,大多数维尔留地区的企业都只位于郊区。
P: 市中心的办公楼缺少当今大多数运营商所需的现代化设施。
C: 为了吸引更多的市区商业,Villieu官员计划建造几座现代化的办公大楼,并为在新市区内租用空间的任何企业降低地方税率。
推测:downtown不繁荣与其他的降低税收不能改变的原因
选项分析:C

4.        31s
P:北方航空每天有数十班航班往返于高度拥挤的贝尔维尔机场。
P: 北方航空的成功取决于经济性和周转速度,因此正计划用空中客车取代其大型飞机,其新颖的空气动力学设计非常省油。
C: Skybus的燃油效率既降低了燃油成本,又减少了加油时间。
推测:油耗降低导致票价降低,可能会使机场更拥挤;可能提高航班数量;变成小飞机减少客流量;造成其他的原因导致的堵塞
选项分析:E

5.        24s
P: 一台新的玉米收割机将使行距仅十五英寸,而不是通常的三十英寸。
P: 紧密种植的玉米单产将降低。
C: 尽管如此,新机器将使玉米种植者每英亩的利润增加一倍。
推测:收获时间短之类的,其他原因可以提高收益
选项分析:A

作者: Lincy123    时间: 2020-5-26 08:29
P:现在的新闻报纸比以前更多杀人犯的故事
C:暴力已经失控,如果没有必要就尽量不出门
Weaken:他因
A)        新闻报纸比以前报道暴力事件更详尽。Correct
B)        国家数据显示暴力事件在所有地方都失控。Strengthen
C)        警察记录显示人们更常受到邻里的暴力。Irrelevant
D)        谋杀在过去占暴力行为的比例更高。Irrelevant
E)        新闻杂志在告知公众暴力行为比以前起着更大作用,irrelevant

HG在urban areas替代MG,但是rural areas两种数量稳定。HG不攻击MG,但是在昆虫聚集的地方,HG不让MG吃昆虫。
Gap:urban areas 和 insects congregate的关系
A)        在城市,G很珍贵因为他们吃虫子。无关
B)        G保护自己的领地不受其他物种侵犯。无关
C)        HG通过船和飞机引进。无关
D)        在有灯光的房子的地方,昆虫喜欢聚集在光周围。Correct
E)        MG都是女性且无性生殖,HG有性生殖。

P:在downtown建造现代办公室大楼,降低入驻企业税
C:吸引企业来downtown
Weaken:office building不是企业搬去suburbs的主要原因
A)        大部分在downtown的企业有长期的租期。无关
B)        在downtown现有的大楼平均有更高的空闲空间比suburbs。无关
C)        Suburbs的税率比downtown优惠后的税率还低。Correct
D)        大部分在downtown租办公室的公司也在suburbs租。无关
E)        建了新楼后,downtown比suburbs有更多新办公室。无关

C:省钱和快的周转率
P:用skybus替换大型飞机,省油,减少换油时间
Weaken:其他cost?
A)        skybus可以让原先要停下来加油的航班直飞。Strengthen
B)        航空燃油费未来几年会下降,无关
C)        虽然skybus减少换油时间,但有些工人失去了工作。无关
D)        没有一个竞争对手考虑买skybus
E)        Skybus的设计让当它起飞时迫使其他飞机延误起飞。Correct

一个新的收割玉米的机器允许15英尺间隔种植,替代通常的30英尺。玉米种植这么紧,平均产量会下降。然而,新机器让玉米种植者每亩利润翻倍,因为
猜测P:多收割的≥单位下降的,成本下降
A)        因为种植紧密,能最小化灌溉成本。Correct
B)        因为种植紧密,玉米长得更高。无关
C)        因为每亩种植的书里那个更多,肥料使用更多。Weaken
D)        因为间隔减少一半,每亩玉米多种1倍
E)        因为种植紧密,玉米地需要更频繁休耕。Weaken





欢迎光临 ChaseDream (https://forum.chasedream.com/) Powered by Discuz! X3.3